Fluid & Electrolytes made incredibly easy

Lakukan tugas rumah & ujian kamu dengan baik sekarang menggunakan Quizwiz!

12. While being treated for hyponatremia, a patient develops iatrogenic hypernatremia. Which treatment is appropriate for resolving this problem? A. Fluid restriction B. Hypotonic fluid administration C. Hypertonic fluid administration D. Diuretic therapy

12. D. Diuretics increase sodium loss in the urine, thereby lowering the serum sodium level.

13. A 35-year-old man with a history of food poisoning and subsequent vomiting complains of weakness, palpitations, abdominal pain, and cramping. His body temperature is 99.6° F (37.6° C). Electrocardiogram results show irregularities. Which imbalance is he most likely to have? A. Hypervolemia B. Hypokalemia C. Acidosis D. Hyperchloremia

13. B. Conditions such as vomiting that lead to loss of gastric acids can cause hypokalemia and alkalosis.

21. A 36-year-old woman with a history of hyperthyroidism has undergone a total thyroidectomy. After surgery, she experiences hypotension, irritability, and circumoral paresthesia. Her surgical wound has well-approximated borders, no bleeding, and minimal swelling. Her speech and breathing are unimpaired. Based on the patient's signs and symptoms, her serum calcium level is likely to be: A. greater than 10.1 mg/dl. B. 10 mg/dl. C. 9 mg/dl. D. 8 mg/dl.

21. D. Hypotension, irritability, and circumoral paresthesia are signs and symptoms of hypocalcemia. Because 8.9 to 10.1 mg/dl is the normal range for total serum calcium levels, 8 mg/dl is the only value here that indicates hypocalcemia.

19. Which finding suggests that a patient has received too much magnesium sulfate? A. Muscle weakness B. Tetany C. Tachycardia D. Hyperreflexia

19. A. Hypermagnesemia causes muscle weakness. Therefore, if a patient develops muscle weakness while receiving magnesium, most likely the dose is too great.

22. A 35-year-old patient with a history of alcohol abuse is admitted with acute pancreatitis. His calcium level on admission is 7.6 mg/dl. Which finding also suggests hypocalcemia? A. Prolonged ST segment on an electrocardiogram (ECG) B. Constipation C. Flaccid reflexes D. Increased cardiac output

22. A. The patient with hypocalcemia may experience diarrhea, hyperactive deep tendon reflexes, a diminished response to digoxin (Lanoxin), decreased cardiac output, prolonged ST segment on an ECG, and a lengthened QT interval, which places the patient at risk for torsades de pointes (polymorphic ventricular tachycardia).

23. A patient with acute hypocalcemia develops torsades de pointes. Which drug is most commonly given to treat acute hypocalcemia? A. Calcium carbonate B. Calcium gluconate C. Calcium chloride D. Calcitonin

23. B. With acute cases of hypocalcemia, I.V. calcium gluconate is usually given. Calcium chloride is a less common alternative.

24. You need to prepare a calcium infusion for a patient with hypocalcemia. You should mix the drug in which solution? A. Normal saline solution B. Dextrose 5% in water C. Half-normal saline solution D. Dextrose 5% in half-normal saline solution

24. B. When preparing a calcium infusion, add calcium to a solution containing dextrose 5% in water. Solutions containing normal saline cause renal calcium loss.

25. A public health nurse in a homeless shelter assesses a 57-year-old man with chronic alcoholism. He has a productive cough and a low-grade fever. He's 5′100 (1.8 m) and weighs 135 lb (61.2 kg). The nurse's nutritional assessment reveals he's malnourished. The patient is admitted to a respiratory isolation room in a community hospital because tuberculosis is suspected. Based on his history of alcohol abuse, you expect his serum phosphorous level to be: A. below normal. B. above normal. C. in the normal range. D. unaffected.

25. A. Patients who abuse alcohol typically have serum phosphorous levels that fall below normal.

26. A patient's phosphorus level is elevated. Which of the following electrolytes should you expect to be decreased? A. Calcium B. Potassium C. Sodium D. Magnesium

26. A. Phosphorus and calcium have an inverse relationship: When the levels of one are increased, the levels of the other are decreased. No such relationship exists between phosphorus and potassium, sodium, or magnesium.

27. You're teaching a patient with hypophosphatemia about the importance of consuming phosphorus-rich foods. You should recommend: A. pumpkin. B. cranberries. C. trout. D. green beans.

27. C. Fish is a food source that's rich in phosphorus, so trout would be helpful to a patient with hypophosphatemia.

20. A patient develops hypermagnesemia. Which intervention is most effective in reducing serum magnesium levels? A. Administer a cation-exchange resin. B. Infuse a bolus of calcium gluconate. C. Increase the volume of I.V. and oral fluids. D. Administer antidiuretic hormone.

20. C. The best method of reducing serum magnesium levels is to increase urinary excretion of magnesium by increasing the patient's fluid intake.

28. A 10-year-old girl who recently returned from traveling abroad complains that she's experienced frequent episodes of diarrhea and weakness for the last 3 days. She's diagnosed with gastroenteritis. Her temperature is 102.4° F (39.1° C), her pulses are weak, and her blood pressure is 76/40 mm Hg. She has poor skin turgor, low urine output, and dry mucous membranes. Serum laboratory studies reveal the child's chloride level to be 88 mEq/L. The direct cause of the child's hypochloremia is most likely: A. fever. B. low urine output. C. diarrhea. D. dry mucous membranes.

28. C. The child's low serum chloride level is probably caused by her diarrhea.

29. A 39-year-old patient is admitted with severe vomiting and abdominal pain. His admission laboratory findings reveal hypochloremia. Which other electrolyte would you expect to be deficient? A. Calcium B. Sodium C. Magnesium D. Phosphorus

29. B. Chloride is a negatively charged ion that has an electrical attraction to sodium. Therefore, if chloride levels become low, so do serum sodium levels.

35. A 27-year-old woman in her 38th week of pregnancy is admitted to the obstetric unit after her bag of water broke at home. You perform a vaginal examination and note that her cervix is 6 cm dilated. You attach the fetal monitor and find that the fetal heart rate is normal. As her labor progresses, she hyperventilates. Which acid-base imbalance is she most likely to experience if she continues to hyperventilate? A. Metabolic acidosis B. Metabolic alkalosis C. Respiratory acidosis D. Respiratory alkalosis

35. D. When a patient hyperventilates, excess carbon dioxide is blown off. This raises the arterial pH above 7.45 causing respiratory alkalosis.

36. A 58-year-old man calls for emergency medical services from his home after he experiences excruciating substernal chest pain. He's rushed to the emergency department where he's given nitroglycerin and morphine for the pain. Electrocardiogram results show changes consistent with an acute anterior wall myocardial infarction (MI). A main complication of an anterior wall MI is heart failure. Which chamber of the heart is most likely to fail in this patient? A. Right atrium B. Right ventricle C. Left atrium D. Left ventricle

36. D. With an anterior wall MI, the left ventricle usually fails, causing heart failure.

45. A 42-year-old man with end-stage AIDS has frequent episodes of watery stool. He's nauseated and refuses to drink fluids. His body temperature is 102° F (38.9° C), his blood pressure is 88/52 mm Hg, and his pulse is 112 beats/minute. Normal saline solution is infusing at 150 ml/hour through a large-bore I.V. catheter. Which type of fluid is normal saline solution? A. Isotonic B. Hypotonic C. Hypertonic D. Colloid

45. A. Normal saline solution is an isotonic crystalloid fluid.

46. A patient's blood volume doesn't improve after the administration of crystalloids. The doctor prescribes a colloid for this patient. Which of the following solutions is a colloid? A. Dextrose 5% in half-normal saline solution B. Hetastarch C. Dextrose 10% in water D. Dextrose 5% in lactated Ringer's solution

46. B. Examples of colloids include albumin, hetastarch, dextran, and plasma protein fraction.

47. An 18-year-old patient with Crohn's disease is unable to tolerate an elemental diet. Total parenteral nutrition (TPN) is indicated when the patient's serum albumin is less than: A. 5 g/dl. B. 4.5 g/dl. C. 4 g/dl. D. 3.5 g/dl.

47. D. TPN is typically indicated when the serum albumin level is less than 3.5 g/dl.

48. A patient receiving total parenteral nutrition (TPN) requires a transfusion of packed red blood cells. Before you begin the transfusion, you should: A. infuse the blood directly into the TPN line. B. start a separate I.V. line for the blood transfusion. C. stop the TPN, infuse the blood at the TPN site, and then restart the TPN. D. use a Y connector and infuse the blood simultaneously with the TPN.

48. B. Blood transfusions shouldn't be infused with TPN; therefore, a separate I.V. line should be secured for the blood transfusion.

49. A patient's postoperative hemoglobin level is 7.9 g/dl. The doctor orders 2 units of packed red blood cells (RBCs) for the patient. By what percentage should this increase the patient's hematocrit? A. 3% B. 6% C. 9% D. 12%

49. B. One unit of packed RBCs will increase hematocrit by 3%; 2 units, by 6%.

5. A 29-year-old patient comes to the emergency department after being involved in a motor vehicle accident. Chest radiography reveals a right pneumothorax. You expect his arterial blood gas results to reflect which of the following? A. His pH is low, PaCO2 is high, and bicarbonate is normal. B. His pH is low, PaCO2 is low, and bicarbonate is low. C. His pH is low, PaCO2 is high, and bicarbonate is low. D. His pH is high, PaCO2 is low, and bicarbonate is low

5. A. In patients with respiratory acidosis, pH is low, PaCO2 is high, and bicarbonate is normal.

50. A patient experiences a transfusion reaction 15 minutes after you begin a blood transfusion. You collect the appropriate laboratory specimens. Laboratory results reveal hemoglobinuria. Which type of reaction has the patient most likely experienced? A. Hemolytic B. Febrile C. Allergic D. Vasogenic

50. A. Hemoglobinuria is a sign of a hemolytic reaction to a blood transfusion and isn't representative of other reaction types.

7. A patient who sustained multiple abdominal injuries in a motor vehicle accident 2 days ago becomes hypotensive. His urine output for the past 4 hours totals 45 ml. The doctor decides to insert a pulmonary artery (PA) catheter. During measurement of PA pressures, what specific information is being obtained when the balloon is wedged in a branch of the PA? A. Left-sided heart function B. Central venous pressure C. Cardiac output D. Right-sided heart function

7. A. When the tip of the PA catheter is wedged in a branch of the pulmonary artery, it measures pressures that reflect left-sided heart function.

8. A patient with Alzheimer's disease is admitted with suspected dehydration after her daughter reports that the patient has refused to drink anything for the past 3 days. The doctor orders several laboratory tests. Which laboratory test result is most expected with dehydration? A. Urine specific gravity of 1.005 B. Serum sodium level of 150 mEq/L C. Hematocrit of 38% D. Serum creatinine level of 0.8 to 1.5 mg/dl

8. B. Because some of the water present in the serum is lost, causing dehydration, the serum sodium level becomes elevated.

9. A 53-year-old homeless person is admitted with dehydration. Which type of I.V. fluid should be avoided when treating this patient? A. Isotonic fluid B. Colloid fluid C. Hypotonic fluid D. Hypertonic fluid

9. D. Dehydration is a hypertonic state; therefore, hypertonic fluid should be avoided because it would worsen the patient's condition. Free water or isotonic or hypotonic fluid would be a safer choice.

4. Infusions of lipid emulsions are useful for promoting: A. wound healing. B. coagulation in bleeding disorders. C. a reduction in inflammation from pancreatitis. D. decreased hemoglobin level and hematocrit in anemia.

Answer: A. Lipid emulsions assist in wound healing, in the production of RBCs, and in prostaglandin synthesis. They should be avoided in patients with acute pancreatitis or a coagulation disorder and used cautiously in patients with acute pancreatitis

3. When teaching your patient with hypomagnesemia about a proper diet, you should recommend that he consume plenty of: A. seafood. B. fruits. C. corn products. D. dairy products.

Answer: A. Magnesium is found in seafood as well as in chocolate; dry beans and peas; meats; nuts; whole grains; and green, leafy vegetables.

4. The doctor prescribes I.V. magnesium sulfate for your patient with hypomagnesemia. Before giving the magnesium preparation, you review the practitioner's order to make sure it specifies the: A. number of grams or milliliters to give. B. number of ampules to give. C. number of vials to give. D. number of uses per vial.

Answer: A. Magnesium sulfate comes in several different concentrations. The practitioner's order should specify the number of grams or milliliters of a particular concentration, plus either the amount of solution to use for dilution or the duration of the infusion.

6. An 83-year-old patient with heart failure develops hypokalemia as a result of her diuretic therapy. You suggest that she increase her dietary intake of potassium. Which foods should she consume? A. Chocolate, orange juice, and bananas B. Canned soups, peas, and milk C. Apples, whole wheat bread, and oatmeal D. Dairy products and whole grains

Answer: A. Major dietary sources of potassium include chocolate, dried fruit, nuts and seeds, oranges, bananas, apricots, cantaloupes, potatoes, mushrooms, tomatoes, and celery.

1. Populations at risk for dehydration include: A. infants. B. adolescents. C. patients with SIADH. D. young adults.

Answer: A. Patients at risk for dehydration are those who either have an impaired thirst mechanism or can't respond to the thirst reflex. Infants fall into this category.

1. When a burn damages cells, you would expect the cells to release the major electrolyte: A. potassium. B. chloride. C. calcium. D. sodium.

Answer: A. Potassium is one of the major electrolytes inside the cell that leaks out into extracellular fluid after a major trauma, such as a burn. This puts the patient at risk for hyperkalemia

2. Respiratory alkalosis can develop from: A. hyperventilation. B. excessive vomiting. C. prolonged use of antacids. D. decreased respiratory rate.

Answer: A. Respiratory alkalosis develops from an excessively rapid respiratory rate— hyperventilation—which causes excessive carbon dioxide elimination.

3. Which of the following is a sign of an allergy to I.V. tubing? A. Shortness of breath B. Dry throat C. Slow, bounding pulse D. Hypertension

Answer: A. Signs and symptoms of an allergic reaction include shortness of breath, rash, and itching.

4. Hypercalcemia would be most likely to develop in: A. a 60-year-old man who has squamous cell carcinoma of the lung. B. an 80-year-old woman who has heart failure and is taking furosemide (Lasix). C. a 25-year-old trauma patient who has received massive blood transfusions. D. a 40-year-old man with hypoalbuminemia.

Answer: A. Squamous cell carcinoma of the lung can lead to hypercalcemia.

3. Many of the signs and symptoms of hypophosphatemia are related to: A. low energy stores. B. hypercalcemia. C. extensive diuresis. D. hypocalcemia.

Answer: A. The body needs phosphorus to make ATP, which provides all the cells— especially muscles—with energy.

2. When the hormone aldosterone is secreted, the kidneys reabsorb: A. sodium. B. potassium. C. magnesium. D. calcium.

Answer: A. The kidneys reabsorb sodium and excrete potassium when aldosterone is secreted

4. In the nephron, most electrolytes are reabsorbed in the: A. proximal tubule. B. glomerulus. C. loop of Henle. D. distal tubule.

Answer: A. The proximal tubule reabsorbs most electrolytes from the filtrate. It also reabsorbs glucose, urea, amino acids, and water.

2. Signs and symptoms of hyponatremia include: A. change in LOC, abdominal cramps, and muscle twitching. B. headache, rapid breathing, and high energy level. C. chest pain, fever, and pericardial rub. D. weight loss, slow pulse, and vision changes.

Answer: A. The signs and symptoms of hyponatremia include change in LOC, abdominal cramps, and muscle twitching. A patient with hyponatremia may also exhibit headache, nausea, coma, blood pressure changes, and tachycardia.

6. Your bedridden patient has these ABG results: pH, 7.5; PaCO2 , 26 mm Hg; bicarbonate, 24 mEq/L. He's dyspneic and has a swollen right calf. The patient most likely is suffering from: A. a pulmonary embolus. B. heart failure. C. dehydration. D. hyperaldosteronism.

Answer: A. Unexplained respiratory alkalosis may mean a pulmonary embolus (in this case, most likely a thrombus in the leg as a result of immobility).

1. The body compensates for chronic respiratory alkalosis by: A. increasing excretion of bicarbonate. B. decreasing excretion of bicarbonate. C. increasing PaCO2 . D. decreasing PaCO2 .

Answer: A. When hypocapnia lasts more than 6 hours, the body develops metabolic acidosis and the kidneys compensate by increasing excretion of bicarbonate and reducing excretion of hydrogen ions. Hydrogen ions return to the blood to decrease pH, causing chemoreceptors in the medulla to decrease the respiratory rate.

6. Hypertonic solutions should be used cautiously in patients with: A. cancer or burns. B. cardiac or renal disease. C. respiratory or GI disease. D. hepatic or renal disease.

Answer: B. A hypertonic solution draws fluids from the intracellular space into the bloodstream. Patients with cardiac or renal disease may be unable to tolerate that extra fluid volume.

4. If your patient has a higher than normal pH (alkalosis), you would expect to also see: A. high PaCO2 and high bicarbonate. B. low PaCO2 and high bicarbonate. C. low bicarbonate and high PaCO2 . D. low PaCO2 and low bicarbonate.

Answer: B. A low PaCO2 means less carbon dioxide (acid) is in the blood, which raises pH. When pH is raised, the bicarbonate level also increases

5. A patient with respiratory failure should limit his intake of: A. protein. B. carbohydrates. C. water. D. sodium.

Answer: B. A patient with respiratory failure should limit carbohydrate intake and increase protein intake because carbohydrate metabolism causes more carbon dioxide production than protein metabolism.

5. Water intoxication can be caused by: A. administering too much hypertonic fluid. B. administering too much hypotonic fluid. C. encouraging fluid intake. D. administering too much isotonic fluid.

Answer: B. Administering too much hypotonic fluid can cause water to shift from the blood vessels into the cells, leading to water intoxication and cellular edema.

1. Albumin affects calcium levels by: A. blocking phosphorus absorption, which prevents calcium excretion. B. binding with calcium, which makes calcium ineffective. C. inhibiting magnesium uptake, which prevents calcium absorption. D. affecting pH level.

Answer: B. Albumin binds with calcium and renders it ineffective.

3. Which feature can help you determine whether a patient has heat exhaustion or heatstroke? A. Temperature higher than 102° F (38.9° C) B. Altered CNS function C. Dehydration D. Elevated liver transaminase levels

Answer: B. Altered CNS function (including seizures, coma, delirium, bizarre behavior, and dilated pupils) is the hallmark of heatstroke. Heat exhaustion causes fatigue and weakness; however, the patient is usually aware. Heat exhaustion, if it is severe and left untreated, may progress to heatstroke.

4. You carefully observe the characteristics of the patient's vomitus and document your finding as brown with fecal odor. This type of vomitus may indicate: A. excessive hydrochloric acid in gastric contents. B. intestinal obstruction. C. obstruction below the pylorus. D. gastric outlet obstruction.

Answer: B. Brown vomitus that has a fecal odor may indicate intestinal obstruction or infarction.

5. In a patient with COPD, the primary imbalance is likely to be: A. respiratory alkalosis. B. respiratory acidosis. C. metabolic alkalosis. D. metabolic acidosis.

Answer: B. COPD results in destruction of the alveoli, thereby decreasing the surface area of the lungs available for gas exchange. With alveolar ventilation decreased, the PaCO2 increases. The carbon dioxide combines with water to form excessive amounts of carbonic acid. The carbonic acid dissociates to release free hydrogen and bicarbonate ions, thereby decreasing the pH (respiratory acidosis).

7. Which of the following disorders causes isovolemic hyponatremia? A. Hyperthyroidism B. SIADH C. Heart failure D. Dementia

Answer: B. Causes of isovolemic hyponatremia include glucocorticoid deficiency, hypothyroidism, renal failure, and SIADH

2. If the level of bicarbonate ions increases, the level of chloride ions: A. increases. B. decreases. C. stays the same. D. isn't affected.

Answer: B. Chloride ions and bicarbonate ions have an inversely proportional relationship. If one level rises, the other level drops

3. If your patient is breathing rapidly, his body is attempting to: A. retain carbon dioxide. B. get rid of excess carbon dioxide. C. improve the buffering ability of bicarbonate. D. produce more carbonic acid.

Answer: B. High carbon dioxide levels in the blood, measured as PaCO2 , cause a drop in pH. Chemoreceptors in the brain sense this decrease and stimulate the lungs to hyperventilate, causing the body to eliminate more carbon dioxide.

1. A patient with hyperkalemia may experience several ECG changes, including: A. flat T waves, a small QRS complex, and normal P waves. B. tall, peaked T waves; a widened QRS complex; and disappearing P waves. C. no T waves, a normal QRS complex, and flattened or misshaped P waves. D. tall, peaked T waves; a normal QRS complex; and disappearing P waves.

Answer: B. High potassium levels may result in disappearing P waves; a widened QRS complex; and tall, peaked T waves because of the effect on cardiac cells.

4. The binding of phosphorus and calcium in a patient with hyperphosphatemia can lead to: A. increased calcium release by the kidneys. B. widespread calcification of tissues. C. decreased calcium uptake by the pituitary gland. D. increased production of PTH.

Answer: B. Hyperphosphatemia results in hypocalcemia. The calcium and phosphorus bind together and are deposited in the tissues, resulting in calcification.

1. Hypertonic solutions cause fluids to move from the: A. interstitial space to the intracellular space. B. intracellular space to the extracellular space. C. extracellular space to the intracellular space. D. intracellular space to the interstitial space.

Answer: B. Hypertonic solutions, because of their increased osmolality, draw fluids out of the cells and into the extracellular space.

5. When a hypotonic crystalloid solution is infused into the bloodstream, it causes the cells to: A. shrink. B. swell. C. release chloride. D. release potassium.

Answer: B. Hypotonic crystalloids are less concentrated than extracellular fluids, so they move from the bloodstream into the cell and cause the cell to expand with fluid.

2. Hypovolemia usually occurs during which major burn phase? A. Fluid remobilization B. Fluid accumulation C. Convalescent D. Acute

Answer: B. Hypovolemia usually occurs during the fluid accumulation phase as fluid moves from the intravascular space to the interstitial space, a process known as thirdspace shift.

3. Of the following options, the first step you should take for a patient with hypovolemic shock is to: A. assess for dehydration. B. administer I.V. fluids. C. insert a urinary catheter. D. prepare for surgery.

Answer: B. Hypovolemic shock is an emergency that requires rapid infusion of I.V. fluids.

7. If administering dopamine to a patient with hypotension proves ineffective, how should you proceed? A. Change to dobutamine. B. Investigate the patient's pH. C. Check the patient's serum potassium level. D. Increase the rate of dopamine infusion.

Answer: B. If you're administering dopamine to a patient and it isn't elevating his blood pressure as you expected, you should investigate the patient's pH. A pH level below 7.1 causes resistance to vasopressor therapy.

6. Your hemodialysis patient needs a laxative. When you see that the practitioner has ordered magnesium citrate, you decide to question the order because: A. this magnesium salt would be too strong for the patient. B. magnesium administration could worsen the patient's condition. C. magnesium citrate must be given orally. D. magnesium citrate can cause nausea and vomiting.

Answer: B. Magnesium citrate is a poor laxative choice for a patient with a renal impairment whose kidneys can't excrete magnesium properly. The patient could develop hypermagnesemia.

3. If your patient's NG tube is attached to suction, you know the patient may develop metabolic alkalosis. You expect that his ABG results will show: A. decreased pH, increased PaCO2 , and decreased bicarbonate. B. increased pH, increased PaCO2 , and increased bicarbonate. C. decreased pH, decreased PaCO2 , and decreased bicarbonate. D. increased pH, decreased PaCO2 , and no change in bicarbonate.

Answer: B. Metabolic acidosis is caused by a loss in hydrogen ion production and a gain in bicarbonate, causing increased pH and bicarbonate levels. If respiratory compensation occurs, PaCO2 will increase above 45 mm Hg.

5. The sodium-potassium pump transports sodium ions: A. into cells. B. out of cells. C. into and out of cells in equal amounts. D. into skeletal muscles.

Answer: B. Normally most abundant outside of cells, sodium tends to diffuse inward. The sodium-potassium pump returns sodium to the extracellular area. Potassium ions tend to diffuse out of the cells and require transport back into cells.

6. Older adults are at increased risk for electrolyte imbalances because, with age, the kidneys have: A. increased glomerular filtration rate. B. fewer functioning nephrons. C. increased ability to concentrate urine. D. increased blood flow.

Answer: B. Older adults are at increased risk for electrolyte imbalances because their kidneys have fewer functioning nephrons, a decreased glomerular filtration rate, and a diminished ability to concentrate urine.

1. PaCO2 level indicates the effectiveness of: A. kidney function. B. lung ventilation. C. phosphate buffers. D. bicarbonate buffers.

Answer: B. PaCO2 reflects how well the respiratory system is helping to maintain acid base balance.

1. If your patient has hyperphosphatemia, he or she may also have the secondary electrolyte disturbance: A. hypermagnesemia. B. hypocalcemia. C. hypernatremia. D. hyperkalemia.

Answer: B. Phosphorus and calcium have an inverse relationship: If serum phosphorus levels are increased, then serum calcium levels are decreased.

5. Potassium is essential for conducting electrical impulses because it causes ions to: A. clump together to generate a current. B. shift in and out of the cell to conduct a current. C. trap sodium inside the cell to maintain a current. D. adhere to each other to create a current.

Answer: B. Potassium in the intracellular fluid causes ions to shift in and out of the cell, which allows electrical impulses to be conducted from cell to cell.

3. You insert an I.V. line and begin fluid resuscitation. The doctor wants you to use the Parkland formula. The patient is a 155-lb (70-kg) male and is estimated at having 50% of his total body surface area burned. What amount of lactated Ringer's solution should you administer over the first 8 hours? A. 700 ml B. 7,000 ml C. 1,400 ml D. 6,000 ml

Answer: B. The Parkland formula is 4 ml × the percentage of total body surface area burned × weight in kilograms. So, 4 ml × 50% × 70 kg = 14,000 ml or 14 Lof lactated Ringer's solution in the first 24 hours. Therefore, you would give 7,000 ml (or half) in the first 8 hours.

5. The laboratory reports the following ABG results for your patient: pH, 7.33; PaCO2 , 40 mm Hg; and bicarbonate, 20 mEq/L. You interpret these results as: A. respiratory acidosis. B. metabolic acidosis. C. respiratory alkalosis. D. metabolic alkalosis.

Answer: B. The patient's pH is low, which indicates acidosis. Because PaCO2 is normal and bicarbonate is low (matching the pH), the primary cause of the problem is metabolic.

1. The pancreas functions as both an exocrine and endocrine gland. Which of these is an example of its exocrine function? A. The pancreas produces hydrochloric acid. B. Amylase is produced in the acinar cells. C. Insulin is produced in the islets of Langerhans. D. The pancreas secretes its enzymes into the stomach.

Answer: B. The production of amylase in the acinar cells is an example of exocrine function.

4. If a patient has a low serum chloride level, what acid-base imbalance would you expect to see? A. Respiratory acidosis B. Metabolic acidosis C. Metabolic alkalosis D. Respiratory alkalosis

Answer: C. A drop in chloride ions causes the body to retain bicarbonate—a base—and results in hypochloremic metabolic alkalosis.

3. Hydrostatic pressure, which pushes fluid out of the capillaries, is opposed by colloid osmotic pressure, which involves: A. reduced renin secretion. B. a decrease in aldosterone. C. the pulling power of albumin to reabsorb water. D. an increase in ADH secretion.

Answer: C. Albumin in capillaries draws water toward it, a process called reabsorption.

2. You're taking care of a patient with obesity-hypoventilation syndrome. You expect to see signs of chronic respiratory acidosis in the patient's ABG results. What do you look for? A. Increasing pH B. Increased PaCO2 C. Increased bicarbonate D. Decreased bicarbonate

Answer: C. As respiratory mechanisms fail, the body compensates by using the increased PaCO2 to excrete hydrogen and to stimulate the kidneys to retain bicarbonate and sodium ions. As a result, more sodium bicarbonate (thus an increased bicarbonate) is available to buffer free hydrogen ions (metabolic alkalosis). Ammonium ions are also excreted to remove hydrogen.

4. Your patient is a 90-year-old male with a history of heart failure. When you make rounds, you notice that an I.V. of normal saline solution was mistakenly given an hour before and has infused 600 ml since then. You should observe this patient for signs of: A. septic shock. B. decreased ICP. C. circulatory overload. D. increased ICP.

Answer: C. Because of his advanced age and cardiac condition, the type of fluid infused, and the infusion rate, the patient is at risk for circulatory overload.

1. In addition to its responsibility for fluid balance, sodium is also responsible for: A. good eyesight and vitamin balance. B. bone structure. C. impulse transmission. D. muscle mass.

Answer: C. Besides its role as the main extracellular cation responsible for regulating fluid balance in the body, sodium is also involved in impulse transmission in nerve and muscle fibers.

4. Which drug class given to treat heart failure has been shown to increase longevity? A. ACE inhibitors B. Nitrates C. Beta-adrenergic blockers D. Digoxin

Answer: C. Beta-adrenergic blocker therapy enhances longevity.

3. Neuromuscular signs and symptoms of hypokalemia include: A. Tourette's syndrome. B. confusion and irritability. C. diminished deep tendon reflexes. D. Parkinsonian-type tremors.

Answer: C. Deep tendon reflexes may be decreased or absent in hypokalemia. Also, leg cramps may occur and respiratory muscles may be paralyzed.

1. When assessing a patient with left-sided heart failure, you would expect to detect: A. distended neck veins. B. edema of the lower extremities. C. dyspnea on exertion. D. hepatomegaly.

Answer: C. Diminished left ventricular function allows blood to pool in the ventricle and atrium and eventually back up into the pulmonary veins and capillaries. As the pulmonary circulation becomes engorged, rising capillary pressure pushes sodium and water into the interstitial space, causing pulmonary edema. Reasons for seeking care include fatigue, exertional dyspnea, orthopnea, weakness, and paroxysmal nocturnal dyspnea.

2. Diuretics affect the kidneys by altering the reabsorption and excretion of: A. water only. B. electrolytes only. C. water and electrolytes. D. other drugs.

Answer: C. Diuretics generally affect how much water and sodium the body excretes. At the same time, other electrolytes such as potassium can also be excreted in urine.

5. The best cooling method for heatstroke is: A. water immersion. B. iced peritoneal lavage. C. evaporative cooling. D. I.V. fluids.

Answer: C. Evaporative cooling, which includes undressing the patient, spraying tepid water on him, and using cool fans to maximize evaporation, is the best cooling method for a patient with heatstroke

2. The most common cause of acute pancreatitis is: A. alcohol. B. eating low-fat foods. C. gallstones. D. pregnancy.

Answer: C. Gallstones are the most common cause of acute pancreatitis.

4. Increased serum sodium levels cause thirst and the release of: A. potassium into the cells. B. fluid into the interstitium. C. ADH into the bloodstream. D. aldosterone into the kidneys.

Answer: C. Higher blood sodium levels prompt the release of ADH from the posterior pituitary gland.

3. If your patient is hypercalcemic, you would expect to: A. administer I.V. sodium bicarbonate. B. administer vitamin D. C. hydrate the patient. D. administer digoxin.

Answer: C. Hydrating a patient with oral or I.V. fluids increases the urine excretion of calcium and helps lower serum calcium levels.

2. Your patient with Crohn's disease develops tremors while receiving TPN. Suspecting she might have hypomagnesemia, you assess her neuromuscular system. You should expect to see: A. Homans' sign. B. elevated serum potassium. C. hyperactive DTRs. D. slowed heart rate.

Answer: C. In a patient with hypomagnesemia, expect to see hyperactive DTRs because hypomagnesemia increases neuromuscular excitability.

4. You're concerned about possible respiratory failure in your newly admitted patient. When administering drug therapy, you should avoid giving him which of the following agents? A. Anticholinergics B. Corticosteroids C. Opioids D. Antihypertensives

Answer: C. Opioids depress the respiratory center of the brain and may hasten the development of respiratory failure.

1. Potassium is responsible for: A. building muscle mass. B. building bone structure and strength. C. maintaining the heartbeat. D. maintaining weight.

Answer: C. Potassium is vital for proper cardiac function because it plays a key role in cardiac muscle contraction and electrical conductivity. Changes in serum potassium level call for early recognition and treatment.

1. Which mechanism accounts for most of the body's heat loss under typical environmental conditions? A. Convection B. Conduction C. Radiation D. Evaporation

Answer: C. Radiation accounts for approximately 65% of the body's heat loss when the outside temperature is less than the body's temperature.

3. In a postoperative patient who has a chloride imbalance, you would also expect to see a change in the electrolyte: A. calcium. B. potassium. C. sodium. D. phosphate.

Answer: C. Sodium and chloride move together through the body, so an imbalance in one usually causes an imbalance in the other.

5. A hallmark ECG characteristic of hyperkalemia is the presence of: A. irregular PR intervals. B. narrowed QRS complexes. C. tall, tented T waves. D. peaked P waves.

Answer: C. Tall, tented T waves are a hallmark of hyperkalemia, a condition that can also lead to heart block, ventricular arrhythmias, and asystole.

1. When the body senses hypoxemia or hypercapnia, the brain's respiratory center responds by A. slowing down the respiratory rate. B. decreasing the heart rate. C. increasing the depth and rate of respirations. D. increasing the heart rate.

Answer: C. The brain's respiratory center initially causes an increase in respiratory rate. It then causes an increase in respiratory depth in an effort to blow off excess carbon dioxide.

3. Laboratory results associated with acute renal failure include: A. incsreased BUN level and decreased serum creatinine level. B. decreased BUN level and increased urine output. C. increased BUN and serum creatinine levels. D. increased BUN level and increased urine output.

Answer: C. The patient with acute renal failure has increased BUN and serum creatinine levels and decreased urine output

2. When preparing to hang a TPN solution, you see an oily layer in the bag. You should: A. gently agitate the solution to disperse the contents. B. hang the solution; the oily layer will disperse in time. C. return the solution to the pharmacy. D. squeeze the bag to mix the solution.

Answer: C. The solution should be clear or pale yellow. An oily layer indicates that the fluid may have been contaminated or improperly prepared and it should be returned to the pharmacy.

7. When administering I.V. potassium for severe hypokalemia, you should: A. avoid infusing the potassium with all other I.V. solutions. B. infuse through a small I.V. catheter. C. verify that the concentration of the solution doesn't exceed 40 mEq/L. D. use the drip method to infuse the potassium.

Answer: C. To prevent or reduce toxic effects, the I.V. infusion concentration shouldn't exceed 40 mEq/L.

8. Before and after you administer sodium bicarbonate, you should flush the I.V. line with: A. heparin. B. sterile water. C. normal saline solution. D. potassium.

Answer: C. You should flush the I.V. line with normal saline solution before and after giving bicarbonate.

4. When assessing a patient with DKA, you detect Kussmaul's respirations. You realize the body is in: A. respiratory alkalosis with compensation. B. respiratory acidosis with compensation. C. metabolic alkalosis with compensation. D. metabolic acidosis with compensation.

Answer: D. A patient with DKA will develop metabolic acidosis. When excess hydrogen can't be buffered, the hydrogen reduces blood pH and stimulates chemoreceptors in the medulla, which in turn increases the respiratory rate (leading to respiratory alkalosis). This mechanism lowers carbon dioxide levels and allows more hydrogen to bind with bicarbonate.

2. For a patient with hyperphosphatemia and renal failure, avoid giving the phosphatebinding antacid: A. aluminum hydroxide. B. calcium carbonate. C. calcium acetate. D. magnesium oxide.

Answer: D. Administering an antacid that contains magnesium to a patient with renal failure can result in hypermagnesemia.

5. Your patient is diagnosed with hypermagnesemia. To treat this imbalance, the practitioner is likely to order: A. magnesium citrate. B. magnesium sulfate diluted in fluids. C. potassium-sparing diuretics. D. oral and I.V. fluids.

Answer: D. Both oral and I.V. fluids may be used to treat hypermagnesemia. By causing diuresis, the fluids promote excretion of excess magnesium by the kidneys.

4. Medications to help treat severe hyperkalemia include: A. methylprednisolone and mannitol. B. mannitol and regular insulin. C. digoxin and diuretics. D. 10% calcium gluconate and regular insulin.

Answer: D. Calcium gluconate helps to stabilize cardiac cell membranes, although it doesn't lower a high potassium level itself. Regular insulin, given with hypertonic dextrose, causes potassium to move into the cells, thus lowering the serum potassium level.

2. After cooling a patient with heatstroke, he begins to shiver. You expect to administer: A. aspirin. B. acetaminophen. C. diazepam. D. chlorpromazine.

Answer: D. Chlorpromazine should be used when a patient is shivering to prevent significant heat production.

4. One sign of hypervolemia is: A. increased urine output. B. clear, watery sputum. C. severe hypertension. D. a rapid, bounding pulse

Answer: D. Excess fluid in the intravascular space causes a rapid, bounding pulse. When hypervolemia progresses, it can fill the lungs with fluid and cause pulmonary edema, as indicated by the presence of pink, frothy sputum.

3. A patient with heart failure is more likely to develop a toxic reaction to digoxin if he has concurrent: A. hyponatremia. B. hyperkalemia. C. hypernatremia. D. hypokalemia.

Answer: D. Hypokalemia, which can occur with diuretic therapy, may lead to digoxin toxicity.

3. Site care and dressing changes for a patient with TPN should be performed at least: A. once a week. B. every other week. C. every day. D. three times a week.

Answer: D. It's recommended that site care and dressing changes be performed three times a week; however, the patient's condition or facility policy may dictate the need for more frequent care.

1. Magnesium is an important electrolyte because it: A. helps control urine volume. B. promotes the production of growth hormone. C. promotes bone growth and strength. D. assists in neuromuscular transmission.

Answer: D. Magnesium acts at the myoneural junction and is vital to nerve and muscle activity.

6. You're teaching a patient with hypernatremia that he needs to restrict daily intake of sodium. Which foods high in sodium should you tell him to avoid? A. Bananas, peaches, and broccoli B. Red meat, chicken, and pork C. Milk, nuts, and liver D. Canned soups, ketchup, and cheese

Answer: D. Major dietary sources of sodium include canned soups and vegetables, cheese, ketchup, processed meats, table salt, salty snack foods, and seafood.

2. Hypocalcemia involves a dysfunction of: A. calcitonin. B. antidiuretic hormone. C. growth hormone. D. PTH.

Answer: D. PTH promotes reabsorption of calcium from the bone to the serum. When secretion of PTH is decreased, hypocalcemia results.

4. The patient with acute pancreatitis may report that his pain decreases: A. when he lies on his stomach. B. after vomiting. C. after eating a large meal. D. when he lies on his side with his knees drawn toward his chest.

Answer: D. Pain caused by acute pancreatitis is commonly relieved when the patient lies on his side with his knees drawn toward his chest.

3. Prolonged respiratory treatment, such as nebulizer use, can lead to which of the following conditions? A. Hypovolemia B. Respiratory alkalosis C. Respiratory acidosis D. Hypervolemia

Answer: D. Prolonged respiratory treatments, such as nebulizer use, can lead to the inhalation and absorption of water vapor, which can lead to hypervolemia.

5. It might be difficult to wean your patient from mechanical ventilation if he has a serum phosphorus level: A. higher than 8 mg/dl. B. between 4.5 and 6.0 mg/dl. C. between 2 and 4 mg/dl. D. lower than 1 mg/dl.

Answer: D. Severe hypophosphatemia can lead to respiratory muscle weakness and impaired contractility of the diaphragm, which compromises the patient's ability to breathe spontaneously

3. The main extracellular cation is: A. calcium. B. potassium. C. bicarbonate. D. sodium.

Answer: D. Sodium is the main extracellular cation. In addition to other functions, it helps regulate fluid balance in the body.

3. Warning signs of hypovolemia associated with GI losses include: A. bradycardia, decreased blood pressure, and decreased urine output. B. tachycardia, increased blood pressure, and increased urine output. C. decreased blood pressure; increased urine output; and warm, flushed skin. D. tachycardia, decreased blood pressure, and decreased urine output.

Answer: D. Tachycardia, decreased blood pressure, and decreased urine output indicate that the patient is experiencing hypovolemia from GI losses.

1. Chloride is primarily produced by the: A. brain. B. kidneys. C. heart. D. stomach.

Answer: D. The chloride ion is largely produced by gastric mucosa and occurs in the form of hydrochloric acid.

2. The kidneys respond to acid-base disturbances by: A. adjusting PaCO2 levels. B. producing phosphate buffers. C. producing protein buffers. D. excreting or reabsorbing hydrogen or bicarbonate.

Answer: D. The kidneys respond to particular acid-base imbalances by excreting or reabsorbing hydrogen or bicarbonate, according to the body's needs.

6. A colleague hands you these ABG results: pH, 7.52; PaCO2 , 47 mm Hg; and bicarbonate, 36 mEq/L. You interpret these results as: A. normal. B. respiratory acidosis. C. respiratory alkalosis with respiratory compensation. D. metabolic alkalosis with respiratory compensation.

Answer: D. The pH is alkalotic. Although both PaCO2 and bicarbonate have changed, the bicarbonate matches the pH. The elevated PaCO2 represents the efforts of the respiratory system to compensate for the alkalosis by retaining carbon dioxide.

5. Patients recovering from acute pancreatitis should eat foods that are: A. low in carbohydrates and high in fats and proteins. B. low in carbohydrates, proteins, and fats. C. high in carbohydrates and fats and low in proteins. D. high in carbohydrates and low in fats and proteins.

Answer: D. The patient recovering from acute pancreatitis should eat foods that are high in carbohydrates and low in fats and proteins.

1. When a severely malnourished patient starts receiving TPN, his laboratory tests show a rapid drop in potassium, magnesium, and phosphorus levels. The findings indicate which of the following conditions? A. Fluid shock B. Hypervolemia C. Hypovolemia D. Refeeding syndrome

Answer: D. These findings are signs of refeeding syndrome.

The desirable amount of fluid intake and loss in adults ranges from 1,500 to 3,500 mL each 24 hours, with most people averaging 2,500 to 2,600 mL per day. A person's intake should normally be approximately balanced by output or fluid loss. The intake-output balance may not always occur in a single 24-hour period but should normally be achieved within 2 to 3 days. Any deviations from normal ranges for a balanced water intake and output should alert the nurse to potential imbalances. T

Electrolytes are substances that are capable of breaking into particles called ions. ion is an atom or molecule carrying an electrical charge.

TPN complications • Electrolyte imbalances • Acid-base imbalances • Heart failure or pulmonary edema • Hyperglycemia • Rebound hypoglycemia • Refeeding syndrome (in severely malnourished patients), which includes a rapid drop in potassium, magnesium, and phosphorus levels

Monitoring • Assess nutritional status and daily weight. • Assess for edema, a sign of fluid overload. • Monitor serum glucose level every 6 hours initially, then once daily. • Monitor electrolyte levels daily at first, then twice weekly. • Monitor protein levels twice weekly. • Monitor BUN and creatinine levels, liver function tests, and nitrogen balance

Potassium imbalances review Potassium basics • Major cation in intracellular fluid (98%) • Affects nerve impulse transmission • Can be disturbed by diseases, injuries, medications, and therapies • Normal range in blood: 3.5 to 5 mEq/L

Potassium balance • Potassium must be ingested daily (40 mEq); the body can't conserve it. • The sodium-potassium pump, renal regulation, and pH level help to maintain balance.

Hyperkalemia • Most dangerous electrolyte disorder • Commonly accompanies metabolic acidosis •Underlying mechanisms: increased intake of potassium, decreased urine excretion of potassium, shift of potassium out of the cells to extracellular fluid, medications • Best clinical indicators: serum potassium levels and ECG tracings • Serum potassium levels exceeding 7 mEq/L: possible serious cardiac arrhythmias leading to cardiac arrest (could be fatal)

Signs and symptoms • Abdominal cramping • Diarrhea • ECG changes (classic sign: tall, tented T wave with early stages and sine wave with late stages) • Hypotension • Irregular pulse rate • Irritability • Muscle weakness, especially in the lower extremities, which may lead to flaccid paralysis • Nausea • Paresthesia • Bradycardia Treatment For mild to moderate cases • Loop diuretics For severe cases • Calcium chloride or gluconate • Bicarbonate • Insulin • Glucose • Kayexalate • Dialysis (CBIGKD ["see big kid"] is the acronym to remember.)

To remember some of the signs and symptoms of hypokalemia, think of the word SUCTION (keep in mind that hypokalemia can be caused by a loss of stomach contents from nasogastric suctioning): Skeletal muscle weakness U wave (ECG changes) Constipation Toxic effects of digoxin (from hypokalemia) Irregular, weak pulse Orthostatic hypotension Numbness (paresthesia).

To help you remember how to treat hyperkalemia, think "see big kid" (CBIGKD): Calcium gluconate Bicarbonate Insulin Glucose Kayexalate Dialysis

You'll recall that starvation may cause a below-normal serum magnesium level. The word STARVED can help you remember some signs and symptoms of hypomagnesemia. Each of its letters stands for a typical clinical finding: Seizures Tetany Anorexia and arrhythmias Rapid heart rate Vomiting Emotional lability DTRs increased.

To remember the signs and symptoms of hypermagnesemia, think RENAL because poor renal excretion is a major cause of this electrolyte imbalance. Here's a letter-by-letter rundown: Reflexes decreased (plus weakness and paralysis) ECG changes (bradycardia) and hypotension Nausea and vomiting Appearance flushed Lethargy (plus drowsiness and coma).

1. A construction worker labors outside in 90° F (32.2° C) temperatures. What hormone will his body release in larger quantities to help him retain water? A. Insulin B. Antidiuretic hormone C. Renin D. Cortisol

1. B. One way the body conserves water is to release more antidiuretic hormone, which reduces diuresis.

10. A 78-year-old patient is admitted with pulmonary edema. The patient is given I.V. morphine. Why? A. To lower his blood pressure B. To promote diuresis C. To slow his breathing D. To remove fluid from his lungs

10. C. Morphine is given to the patient with pulmonary edema because it relieves air hunger and dilates blood vessels, which in turn reduces pulmonary congestion and the amount of blood that returns to the heart.

11. A patient diagnosed with lung cancer develops syndrome of inappropriate antidiuretic hormone, which puts him at risk for hyponatremia. Which serum sodium level indicates hyponatremia? A. 128 mEq/L B. 135 mEq/L C. 142 mEq/L D. 150 mEq/L

11. A. Normal serum sodium level is 135 to 145 mEq/L. A serum sodium level less than 135 mEq/Lindicates hyponatremia.

14. A 65-year-old patient receives daily doses of furosemide (Lasix) and digoxin (Lanoxin) for treatment of heart failure. His serum potassium level is 3.1 mEq/L. Which associated electrocardiogram (ECG) changes would you expect? A. Peaked T wave B. Depressed ST segment C. Narrow QRS complexes D. Absent P waves

14. B. Hypokalemia causes various ECG changes, including a flattened or inverted T wave, a depressed ST segment, and a characteristic U wave.

15. As part of a patient's treatment for hypokalemia, the doctor prescribes I.V. potassium supplementation. At which rate should it be administered? A. 5 mEq/hour B. 10 mEq/hour C. 15 mEq/hour D. 20 mEq/hour

15. B. When supplemental potassium is given by I.V. infusion, it should be administered at a rate of 10 mEq/hour.

16. A patient with a history of systemic lupus erythematosus develops hyperkalemia. The doctor prescribes sodium polystyrene sulfonate (Kayexalate) to reduce the patient's serum potassium level. This drug works by: A. forcing potassium into the cells. B. promoting renal excretion of potassium. C. pulling potassium out of the bowel for excretion. D. pulling potassium into the bowel for excretion.

16. D. Sodium polystyrene sulfonate is a cation-exchange resin that causes potassium to move out of the blood into the intestines. It's then excreted in the stool.

17. A 28-year-old patient is seen in the obstetrics clinic with a blood pressure of 220/130 mm Hg and abnormal reflexes. The nurse-midwife caring for her suspects preeclampsia. A urinalysis for protein is ordered immediately, and proteinuria is detected. The patient is transported to the obstetric unit in the medical center. On admission, the nurse assesses the patient's deep tendon reflexes as 4+. This value means the reflexes are: A. normal and active. B. present but diminished. C. slow to respond. D. hyperactive.

17. D. Deep tendon reflexes are graded on a 0 to 4+ scale. 0 is absent, 1+ is present but diminished, 2+ is normal, 3+ is increased but not necessarily abnormal, and 4+ is hyperactive.

18. Which intervention is most appropriate for the patient receiving a continuous magnesium sulfate infusion? A. Insert an indwelling urinary catheter. B. Attach the patient to a continuous cardiac monitor. C. Administer calcium gluconate every 4 hours. D. Perform neurologic checks every 2 hours.

18. B. Magnesium affects cardiac function and can cause arrhythmias. Therefore, any patient receiving a magnesium sulfate infusion should be on continuous cardiac monitoring.

2. A postoperative patient is ordered an I.V. solution of dextrose 5% in normal saline solution. What type of fluid is this solution? A. Hypertonic B. Hypotonic C. Isotonic D. Colloid

2. A. A solution of dextrose 5% in normal saline is considered hypertonic because its osmolality is 560 mOsm/L.

32. A 23-year-old patient is admitted with diabetic ketoacidosis. Which value from the arterial blood gas analysis supports the diagnosis? A. pH: 7.48 B. PaCO2 : 48 mm Hg C. Bicarbonate: 28 mEq/L D. Anion gap: 17 mEq/L

32. D. Metabolic acidosis causes the anion gap to be greater than 14 mEq/L. With metabolic acidosis, pH will be less than 7.35, bicarbonate will be less than 22 mEq/L, and PaCO2 will typically be unaffected.

33. You're caring for a 33-year-old patient who developed Guillain-Barré syndrome 1 week after contracting an upper respiratory infection. This places the patient at risk for which acid-base imbalance? A. Metabolic acidosis B. Metabolic alkalosis C. Respiratory acidosis D. Respiratory alkalosis

33. C. In certain neuromuscular diseases, such as Guillain-Barré syndrome, the respiratory muscles fail to respond properly to the respiratory drive, leading to respiratory acidosis.

34. A patient with a history of heart failure calls you to her room because she's short of breath. You assess her and find that her heart failure is worsening. Which type of fluid volume excess is the patient experiencing because of her heart failure? A. Intravascular B. Extracellular C. Intracellular D. Interstitial

34. B. Because the heart doesn't pump effectively in a patient with heart failure, fluid imbalances develop. The most common fluid imbalance associated with heart failure is extracellular volume excess. This results from the heart's failure to propel blood forward, consequent vascular pooling, and the sodium and water reabsorption triggered by the renin-angiotensin-aldosterone system.

3. You're teaching a group of athletes how to prevent excessive fluid loss. You should tell them to consume fluids when they: A. experience dry mouth. B. feel light-headed or dizzy. C. are thirsty. D. sweat.

3. C. The simplest mechanism for maintaining fluid balance is the thirst mechanism. When an individual senses thirst, he should drink to replace lost fluid.

30. A 62-year-old patient who underwent a partial gastrectomy 2 days ago develops hypochloremia. This places the patient at risk for: A. respiratory acidosis. B. respiratory alkalosis. C. metabolic acidosis. D. metabolic alkalosis.

30. D. To compensate for a chloride loss (hypochloremia), the kidneys retain bicarbonate. The accumulation of excess bicarbonate in extracellular fluid can raise the arterial pH above 7.45, causing metabolic alkalosis.

31. A 16-year-old male with a recent history of weight loss, increased appetite, and urinary frequency is seen in the clinic. He complains of weakness and syncope. On initial observation, the nurse notes that his skin and mucous membranes are dry and that his eyeballs appear sunken. The teen's mother reports that he gets up a lot at night to go to the bathroom. His capillary blood glucose measurement is 480 mg/dl. Which acid-base imbalance should you suspect? A. Metabolic acidosis B. Metabolic alkalosis C. Respiratory acidosis D. Respiratory alkalosis

31. A. The patient has signs and symptoms of type 1 diabetes mellitus. Because of the accumulation of metabolic wastes (e.g., ketones), type 1 diabetes mellitus is most commonly associated with metabolic acidosis.

37. A 74-year-old man with a 3-day history of worsening chronic obstructive pulmonary disease is hospitalized. His breathing is labored, breath sounds are congested with rhonchi throughout, and his SaO2 (as measured by pulse oximetry) is 89%. He's placed on a 35% aerosol mask, and blood is drawn for arterial blood gas analysis. The results are pH, 7.33; PaO2 , 68 mm Hg; PaCO2 , 53 mm Hg; and bicarbonate, 18 mEq/L. Which acid-base imbalance does the patient most likely have? A. Metabolic acidosis B. Metabolic alkalosis C. Respiratory acidosis D. Respiratory alkalosis

37. C. When a patient's PaCO2 is elevated, carbonic acid is retained, leading to acidosis. Because the acidosis is respiratory in origin, the patient most likely has respiratory acidosis.

38. You're caring for a 54-year-old patient who has smoked two packs of cigarettes per day for the past 35 years. He's been admitted with worsening chronic obstructive pulmonary disease (COPD). Why is it important for supplemental oxygen to be carefully monitored in this patient? A. Increasing the PaO2 beyond what's needed will lead to oxygen toxicity. B. High oxygen levels will promote microbial growth in the patient's lungs. C. Increased PaO2 levels can depress the drive to breathe in patients with COPD. D. Increased PaO2 levels can elevate the drive to breathe in patients with COPD.

38. C. Increased PaO2 can depress the patient's drive to breathe, which is largely driven by hypoxemia.

39. A patient returned from the postanesthesia care unit with a nasogastric (NG) tube in place. The doctor's order states irrigate NG tube q4h. Which solution is the best irrigant? A. Saline solution B. Distilled water C. Tap water D. Sterile water

39. A. The best solution for gastric irrigation is an isotonic solution such as saline solution.

4. A patient with hyponatremia caused by diabetes insipidus requires I.V. fluid replacement. Which I.V. fluid would provide the greatest concentration of sodium replacement if the patient were to develop a subnormal serum sodium level? A. Dextrose 5% in water B. Half-normal saline solution C. Ringer's solution D. Dextrose 5% in lactated Ringer's solution

4. C. Ringer's solution contains 147 mEq of sodium per liter. Half-normal saline solution contains 77 mEq/L. Dextrose 5% in water contains no sodium. Dextrose 5% in lactated Ringer's solution contains 130 mEq/L.

40. A 66-year-old woman who survived a cardiac arrest was admitted to the intensive care unit. She experienced a prolonged episode of hypotension and is now in acute renal failure. Frequent electrolyte levels are ordered. Hemodialysis is scheduled to begin within 24 hours. Which type of renal failure did the patient experience? A. Intrarenal B. Prerenal C. Postrenal D. Renal

40. B. The patient's renal failure was due to hypotension, which is a prerenal cause. Prerenal causes are those conditions outside of the kidneys that diminish blood flow to the kidneys.

41. A patient with a history of hypertension develops chronic renal failure. What should you expect the glomerular filtration rate (GFR) to be? A. 10 to 20 ml/minute B. 20 to 40 ml/minute C. 40 to 60 ml/minute D. Greater than 60 ml/minute

41. A. Renal failure occurs when the GFR is 10 to 20 ml/minute. A rate of 40 to 70 ml/minute indicates renal reserve; 20 to 40 ml/minute, renal insufficiency; and less than 10 ml/minute, end-stage renal disease.

42. A patient who sustained massive internal injuries in a motor vehicle accident becomes hypotensive and develops acute renal failure. Which acid-base imbalance is this patient most likely to experience? A. Respiratory acidosis B. Respiratory alkalosis C. Metabolic acidosis D. Metabolic alkalosis

42. C. As the kidneys lose their ability to excrete hydrogen ions, there's a buildup of hydrogen, which leads to metabolic acidosis.

43. A patient received burn injuries 48 hours ago. He's entering the second phase of burn injury. What physiologic changes can be expected? A. Edema development B. Increased blood volume C. Decreased hemoglobin level D. Profuse urination

43. D. The second phase of the burn injury, known as the remobilization phase, starts about 48 hours after the initial injury. During this phase, fluid shifts back to the vascular compartment. Edema at the burn site decreases and blood flow to kidneys increases, which increases diuresis.

44. A fireman sustains burns while fighting an apartment fire. He receives fluid resuscitation using the Parkland formula. Which type of fluid is used? A. Normal saline solution B. Half-normal saline solution C. Lactated Ringer's solution D. Dextrose 5% in lactated Ringer's solution

44. C. The Parkland formula, which is widely used for burn resuscitation, uses lactated Ringer's solution.

51. You're caring for a patient with a decreased calcium level of 7.4 mg/dl. Which treatment would you expect to provide? (Select all that apply.) A. I.V. calcium gluconate or I.V. calcium chloride B. Loop diuretics C. Magnesium supplements D. Vitamin D E. Normal saline solution F. Hemodialysis or peritoneal dialysis

51. A, C, D. Treatment of hypocalcemia focuses on correcting the imbalance as quickly as possible. I.V. calcium gluconate or I.V. calcium chloride replaces calcium levels. Because hypocalcemia may not be corrected by calcium therapy alone, expect to give magnesium supplements as well. Also, vitamin D supplements may be ordered to facilitate calcium's absorption in the gastrointestinal tract.

52. You're caring for a patient with a suspected overdose of magnesium-containing antacids and laxatives. On admission, her laboratory values for magnesium were greater than 3 mg/dl. Which signs and symptoms would you expect to see? (Select all that apply.) A. Flushing B. Hypertension C. Hypotension D. Seizures E. Nausea and vomiting F. Bradycardia

52. A, C, E, F. Too much magnesium causes vasodilation and irregular heart muscle contractions, which decrease the blood pressure and slow the heart rate. It may also cause nausea and vomiting, facial flushing, and feelings of warmth.

53. A newly admitted patient is being treated for acute pancreatitis. Which electrolyte disorder may be noted? A. Hypokalemia B. Hypercalcemia C. Hypermagnesemia D. Hypophosphatemia

53. A. Hypokalemia may be caused by severe vomiting and diarrhea in acute pancreatitis that results in potassium loss.

54. The major extracellular anion is: A. potassium. B. sodium. C. chloride. D. magnesium.

54. C. Chloride regulates osmotic pressure between compartments and forms hydrochloric acid in the stomach.

55. Water composes what percentage of total body weight, depending on the amount of fat present? A. 20% to 30% B. 20% to 40% C. 30% to 50% D. 45% to 75%

55. D. Water weight is highest during infancy, constituting up to 75% of total body weight. It begins declining with age due to the amount of increased body fat. In an older adult, body water content is 45% to 55% of body weight.

56. A patient arrives at the emergency department with gastroenteritis caused by dehydration. The admitting nurse records that the patient has been experiencing vomiting and diarrhea for the past 3 days. The doctor orders a continuous I.V. infusion. Which I.V. solution is best to administer? A. Dextrose 5% in 0.45% saline solution B. Dextrose 5% in lactated Ringer's solution C. 0.45% saline solution D. Lactated Ringer's solution

56. D. Lactated Ringer's solution is the infusion of choice for acute volume expansion. It contains a small amount of potassium along with lactate, a form of lactic acid that's metabolized by the liver to form bicarbonate, which helps buffer the blood against the effects of acidosis.

57. A 65-year-old woman is admitted to the emergency department after vomiting excessively at home. After checking the patient's arterial blood gas (ABG) levels, the doctor diagnoses severe dehydration. Using the ABG results as guide, which acid-base imbalance would you expect the patient to have? A. Respiratory acidosis B. Metabolic alkalosis C. Respiratory alkalosis D. Metabolic acidosis

57. B. Metabolic alkalosis causes an increase in bicarbonate level, resulting in a nonrespiratory loss of acid.

58. A malnourished 55-year-old patient with a history of alcohol abuse arrives in the emergency department complaining of muscle weakness and cramps. Electrocardiogram tracings show evidence of arrhythmias, and laboratory tests reveal hypomagnesemia. Which electrolytes are typically depleted with magnesium deficiency? A. Calcium and phosphorus B. Potassium and phosphorus C. Potassium and chloride D. Chloride and calcium

58. B. Malnutrition, diarrhea, and diuretic use commonly cause hypomagnesemia. Loss of potassium and phosphorus from skeletal muscles typically results in muscle weakness, cramps, and arrhythmias.

59. The doctor orders tap water enemas until clear for a patient scheduled for a colonoscopy in the morning. The nurse is aware that after three such enemas, electrolyte imbalances are likely to occur. Signs of which imbalance should cause the most concern? A. Hypocalcemia B. Hypercalcemia C. Hypernatremia D. Hypokalemia

59. D. Tap water enemas can cause a fluid volume deficit, which consequently decreases sodium and potassium levels. This can lead to water intoxication, a potentially life- threatening condition.

6. A patient is transferred to the intensive care unit in septic shock. Arterial blood gas results show that the patient is acidotic. You expect the anion gap to be: A. 0 to 4 mEq/L. B. 4 to 8 mEq/L. C. 8 to 14 mEq/L. D. greater than 14 mEq/L.

6. D. Patients who are in an acidotic state typically have higher than normal amounts of organic acids, which leads to an elevated anion gap (> 14 mEq/L).

60. Electrolytes are made up of: A. glucose, bases, and salts. B. lipids, acids, and bases. C. bases, acids, and salts. D. salts, glucose, and lipids.

60. C. Bases, acids, and salts dissociate into ions when in a watery solution

4. During the fluid remobilization phase of a patient with burn injuries, the nurse would expect to see signs of which electrolyte imbalance? A. Hypokalemia B. Hyperkalemia C. Hypernatremia D. Hypovolemia

Answer: A. Hypokalemia occurs in the fluid remobilization (diuresis) phase as potassium shifts from the extracellular fluid back into the cells.

1. Which of the following fluid and electrolyte imbalances can occur with excessive GI fluid loss? A. Hypomagnesemia, hypermagnesemia, and hyponatremia B. Hypomagnesemia, hypernatremia, and hyperchloremia C. Hypervolemia, hyponatremia, and hypernatremia D. Hypervolemia, hypomagnesemia, and hyperkalemia

Answer: A. Hypomagnesemia, hypermagnesemia, and hyponatremia—and others—may occur with varying types of GI fluid loss.

2. Hypotonic fluids shouldn't be used for a patient with: A. increased ICP. B. DKA whose blood glucose level is 200 mg/dl or more. C. blood loss as a result of trauma. D. water replacement.

Answer: A. Hypotonic fluids cause swelling of the cells and can further increase ICP.

8. Drugs that may cause high sodium levels include: A. antacids. B. diuretics. C. antipsychotics. D. antidepressants.

Answer: A. If taken on a regular basis, antacids with sodium bicarbonate may cause high sodium levels.

The ABCs of ABGs Normal ABG levels for adult patients. pH 7.35 to 7.45 PaCO2 35 to 45 mm Hg Bicarbonate 22 to 26 mEq/L

Acid-base imbalances review Acid-base basics • Acid-base balance depends on the regulation of free hydrogen ions. • Balance is maintained by chemical buffers, respiratory reactions, and kidney reactions. • ABG analysis is the major diagnostic tool for evaluating acid-base states: - pH: determines the extent of acidity or alkalinity, both of which are measured - PaCO2 : reflects the adequacy of ventilation by the lungs - Bicarbonate level: reflects the activity of the kidneys in retaining or excreting bicarbonate. • If hydrogen ion concentration increases, pH decreases (acidosis). • If hydrogen ion concentration decreases, pH increases (alkalosis).

3. Which of these imbalances typically occurs in acute pancreatitis? A. Hypovolemia B. Hypercalcemia C. Hypernatremia D. Hypermagnesemia

Answer: A. In acute pancreatitis, fluid shifting from the intravascular space into the interstitial spaces and retroperitoneum causes hypovolemia.

4. When a person's blood pressure drops, the kidneys respond by: A. secreting renin. B. producing aldosterone. C. slowing the release of ADH. D. secreting ANP.

Answer: A. Juxtaglomerular cells in the kidneys secrete renin in response to low blood flow or a low sodium level. The eventual effect of renin secretion is an increase in blood pressure.

2. Which of the following is the optimal diet for a patient with renal failure? A. High-calorie, low-protein, low-sodium, low-potassium B. High-calorie, high-protein, high-sodium, high-potassium C. Low-calorie, high-protein, low-sodium, low-potassium D. High-calorie, low-protein, low-sodium, high-potassium

Answer: A. A high-calorie, low-protein, low-sodium, and low-potassium diet is the optimal diet for meeting the metabolic and nutritional requirements of a patient with renal failure.

3. The minimum daily requirement of sodium for an average adult is: A. 2 g. B. 4 g. C. 5 g. D. 8 g.

Answer: A. Although the minimum daily requirement is 2 g, most people consume more than 6 g/day.

5. Giving a hypertonic I.V. solution to a patient may cause too much fluid to be: A. pulled from the cells into the bloodstream, which may cause the cells to shrink. B. pulled out of the bloodstream into the cells. C. pushed out of the bloodstream into the extravascular spaces. D. pulled from the cells into the bloodstream, which may cause the cells to increase in size.

Answer: A. Because the concentration of solutes in the I.V. solution is greater than the concentration of solutes in the patient's blood, a hypertonic solution may cause fluid to be pulled from the cells into the bloodstream, causing the cells to shrink

1. If you were walking across the Sahara Desert with an empty canteen, the amount of ADH secreted would most likely: A. increase. B. decrease. C. stay the same. D. have no effect.

Answer: A. Because your body would probably be dehydrated, it would try to retain as much fluid as possible. To retain fluid, ADH secretion increases.

2. Checking for orthostatic hypotension allows the nurse to detect early signs of: A. hypovolemia. B. low serum osmolality. C. high serum osmolality. D. hypervolemia.

Answer: A. Changes in blood pressure—which can result in orthostatic hypotension— and pulse are two initial changes seen with hypovolemia.

5. You're told during shift report that your patient has a positive Chvostek's sign. You would expect his laboratory test results to reveal: A. a total serum calcium level below 8.9 mg/dl. B. a total serum calcium level above 10.1 mg/dl. C. an ionized calcium level above 5.3 mg/dl. D. an ionized calcium level between 4.4 and 5.3 mg/dl.

Answer: A. Chvostek's and Trousseau's signs are associated with hypocalcemia. A total serum calcium level below 8.9 mg/dl confirms the presence of that condition.

5. Deep, rapid breathing may indicate a: A. serum chloride level greater than 108 mEq/L. B. serum chloride less than 98 mEq/L. C. pH greater than 7.45. D. normal chloride levels.

Answer: A. Deep, rapid breathing, or Kussmaul's respirations, is the body's attempt to blow off excess acid in the form of carbon dioxide. When this occurs, suspect metabolic acidosis, a condition associated with a serum chloride level greater than 108 mEq/L.

1. During the fluid accumulation phase of a major burn injury, fluids shift from the: A. intravascular space to the interstitial space. B. interstitial space to the intravascular space. C. intracellular space to the interstitial space. D. intravascular space to the intracellular space.

Answer: A. During the fluid accumulation phase, fluids shift from the intravascular space to the interstitial space.

2. The most common fluid imbalance associated with heart failure is: A. hypervolemia. B. hypovolemia. C. hyperkalemia. D. hypokalemia.

Answer: A. Extracellular fluid volume excess results from the heart's failure to propel blood forward, which causes vascular pooling, and from the sodium and water reabsorption triggered by the renin-angiotensin-aldosterone system.

2. A patient with fluid losses from the upper GI tract is likely to suffer from which of the following imbalances? A. Metabolic alkalosis B. Metabolic acidosis C. Respiratory acidosis D. Metabolic acidosis

Answer: A. Fluid losses from the upper GI tract can result in metabolic alkalosis; losses from the lower GI tract can result in metabolic acidosis.

2. If you placed two containers next to each other, separated only by a semipermeable membrane, and the solution in one container was hypotonic relative to the other, fluid in the hypotonic container would: A. move out of the hypotonic container into the other. B. pull fluid from the other container into the hypotonic container. C. cause osmosis to occur. D. stay unchanged within the hypotonic container.

Answer: A. Fluid would move out of the hypotonic container into the other container to equalize the concentration of fluid within the two containers. Osmosis occurs when fluid moves from an area with more fluid to an area with less fluid.

4. When treating a patient with heatstroke, you need to monitor temperature continuously to make sure that it doesn't fall below: A. 101° F (38.3° C). B. 98.6° F (37° C). C. 104° F (40° C). D. 99° F (37.2° C).

Answer: A. For patients with heatstroke, a rapid decrease in body temperature below 101° F (38.3° C) can cause hypothermia.

To remember how fluids shift following a burn injury (the burn phases), think of an ARC: Accumulation phase—fluids shift from the vascular compartment into the interstitial spaces. Remobilization phase—fluid moves back to the vascular compartment. Convalescent phase—major fluid shifts have been resolved and healing can begin.

Burn basics • Usually cause major changes in the body's fluid and electrolyte balance, which may change over time as the initial injury progresses • Types: thermal, electrical, chemical, and radiation Classification of burns • First-degree: superficial burn that affects the epidermis; fluid and electrolyte balance isn't affected • Second-degree: partial-thickness burn that affects the epidermis and dermis; fluid and electrolyte imbalances occur with burns that cover significant areas of the body • Third-degree: full-thickness burn that affects the epidermis, dermis, and tissues below the dermis; carries the greatest risk of fluid and electrolyte imbalances • Fourth-degree: deep full-thickness burns that extend to the muscle; prognosis is poor

Heat rash • Develops from excessive sweating during hot weather • Usually found on the neck or upper torso or in skin folds • Looks like red pimples or tiny blisters Heat cramps • Caused by deficiency of water and sodium • Generally attributed to dehydration and poor muscle conditioning Heat exhaustion • Caused by heat and fluid loss from excessive sweating without fluid replacement • Can result in circulatory collapse if not treated promptly Heat syncope (fainting) • Occurs when a patient stands up quickly or has been standing for a prolonged period of time • Dehydration is often to blame for heat syncope! Heatstroke • Caused by rising body temperature • Leads to damage of internal organs • Considered a medical emergency

Causes • Fever • Infection • Dehydration • Burns • Cardiovascular disease • Skin diseases • Sweat gland dysfunction • Diabetes • Hyperthyroidism • Pheochromocytoma • Obesity • High temperatures or humidity • Lack of acclimatization • Drugs (such as amphetamines) and alcohol Treatment • Institute cooling measures to lower body temperature. • Remove or loosen clothing. • Institute rehydration measures and replace sodium and potassium losses. • Treat heatstroke as a medical emergency by initiating the ABCs of life support if needed. • For heatstroke, monitor temperature, intake, output, and cardiac status. • Patients experiencing heatstroke may require administration of benzodiazepines to control seizures, dobutamine I.V. to correct cardiogenic shock, chlorpromazine I.V. to reduce shivering, or mannitol I.V. to maintain urine output.

Heart failure review Heart failure • Clinical syndrome of myocardial dysfunction that causes diminished cardiac output • Occurs when the heart can't pump enough blood to meet the body's metabolic needs Left-sided heart failure • Typically leads to and is the main cause of right-sided heart failure •Causes pulmonary edema, hypoxia, and hypercapnia •Clinical symptoms: fatigue; weakness; orthopnea; exertional dyspnea; pulmonary edema; paroxysmal nocturnal dyspnea; tachycardia; third and fourth heart sounds; tachypnea; shortness of breath; oliguria; and coughing with pink, frothy sputum Right-sided heart failure • Causes enlargement of the abdominal organs and tissue edema • Clinical signs and symptoms: venous engorgement, edema, weight gain, anorexia, nausea, cyanosis of nail beds, cool and clammy skin, chest tightness, palpitations, neck vein distention and rigidity, cardiac arrest, and hepatomegaly

Causes • MI • Myocarditis • Myocardial fibrosis • Ventricular aneurysm • Ventricular overload as a result of aortic insufficiency or ventricular septal defect • Systemic or pulmonary hypertension as a result of aortic or pulmonic stenosis • Restricted ventricular diastolic filling triggered by constrictive pericarditis or cardiomyopathy, tachyarrhythmias, cardiac tamponade, or mitral or aortic stenosis Imbalances caused by heart failure • Hypervolemia or hypovolemia • Hyperkalemia or hypokalemia • Hypochloremia, hypomagnesemia, and hyponatremia • Metabolic acidosis or alkalosis • Respiratory acidosis or alkalosis

Chloride imbalances review Chloride basics • Most abundant anion in extracellular fluid • Moves with sodium and potassium • Helps maintain serum osmolality and water balance • Can combine with sodium, helping the choroid plexus to attract water and form CSF • Is also found in bile and gastric and pancreatic juices in the form of hydrochloric acid, which aids digestion and enzyme activation • Helps maintain acid-base balance and carbon dioxide transport in red blood cells • Normal range: 98 to 108 mEq/L

Chloride balance • Regulation depends on intake and excretion of chloride and reabsorption of chloride ions by the kidneys. • Chloride is absorbed in the intestines; GI disorders may affect balance. • Sodium levels are closely linked with chloride and affected by aldosterone secretion. • The inverse relationship to bicarbonate affects acid-base balance.

Fluid remobilization phase • Begins about 48 hours after the initial burn • Causes fluid to shift back into the vascular compartment • Causes such fluid and electrolyte imbalances as: -Hypokalemia—can develop as potassium shifts from extracellular fluid back into the cells; usually occurs 4 to 5 days after a major burn - Hypervolemia—can occur as fluid shifts back to the vascular compartment; may result from giving too much I.V. fluid - Hyponatremia—may occur when sodium is lost during diuresis - Metabolic acidosis—occurs when loss of sodium results in the depletion of bicarbonate

Convalescent phase • Begins after first two phases have been resolved • May cause further fluid and electrolyte imbalances as a result of inadequate dietary intake Treatment • Depends on the severity of the burn • For severe burns, airway, breathing, and circulation are priorities • Involves rehydration with fluid resuscitation • Requires monitoring of urine output with catheter • May require pain relief measures • Involves wound care

How atrial natriuretic peptide works When blood volume and blood pressure rise and begin to stretch the atria, the heart's ANP shuts off the renin-angiotensin-aldosterone system, which stabilizes blood volume and blood pressure. This powerful hormone: • suppresses serum renin levels • decreases aldosterone release from the adrenal glands • increases glomerular filtration, which increases urine excretion of sodium and water • decreases ADH release from the posterior pituitary gland • reduces vascular resistance by causing vasodilation.

Dehydration in elderly people The signs and symptoms of dehydration may be different in older adults. For example, they might include: • confusion • subnormal temperature • tachycardia • pinched facial expression.

Body Fluid Compartments (2 parts) Intracellular fluid (ICF) is the fluid within cells, -70% of the total body water or 40% of the adult's body weight. Extracellular fluid (ECF) is all the fluid outside the cells, -30% of the total body water or 20% of the adult's body weight.

ECF(extracellular fluid) includes two major areas: Intravascular Interstitial Transcellular Intravascular fluid, or plasma, is the liquid component of the blood (i.e., fluid found within the vascular system). Interstitial fluid is the fluid that surrounds tissue cells and includes lymph. Transcellular fluids include cerebrospinal, pericardial, synovial, intraocular, and pleural fluids, as well as sweat and digestive secretions. To help you remember which fluid belongs in which compartment, keep in mind that inter means between (as in interval—between two events) and intra means within or inside (as in intravenous —inside a vein)

Fluid types • Isotonic—equally concentrated with other solutions • Hypotonic—less concentrated than other solutions • Hypertonic—more concentrated than other solutions

Fluid movement • Diffusion—form of passive transport (no energy is required) that moves solutes from an area of higher concentration to an area of lower concentration, resulting in an equal distribution of solutes between the two areas • Active transport—uses ATP to move solutes from an area of low concentration to an area of higher concentration; example: sodium-potassium pump • Osmosis—passive movement of fluid across a membrane from an area of lower solute concentration to an area of higher solute concentration; stops when both sides have an equal solute concentration • Capillary filtration—movement of fluid through capillary walls through hydrostatic pressure; balanced by plasma colloid osmotic pressure from albumin that causes reabsorption of fluid and solutes

Fluid is lost from the body through sensible and insensible losses. Sensible losses can be measured and include fluid lost during urination, defecation, and wounds. Insensible losses cannot be measured or seen and include fluid lost from evaporation through the skin and as water vapor from the lungs during respiration.

Fluid output averages 2,500 to 2,900 mL per day (average 2,600 mL), with approximately 1,500 mL as urine from the kidneys, 600 mL fluid loss from the skin, 300 mL from the lungs, and 200 mL in feces via the GI tract.

Potassium (K+): (major cation of ICF) normal serum concentration of potassium: 3.5-5.0 mEq/L

Functions -Controls intracellular osmolality -Regulator of cellular enzyme activity -Role in the transmission of electrical impulses in nerve, heart, skeletal, intestinal, and lung tissue; Regulation of acid-base balance by cellular exchange with H+ Sources and Losses -Adequate quantities via a well-balanced diet -Leading food sources: fruits and vegetables, dried peas and beans, whole grains, milk, meats -Lost via kidneys, stool, sweat, emesis Regulation -Regulated by aldosterone -Eliminated by the kidneys (no effective method of conserving potassium) -Additional regulation via transcellular shift between the ICF and ECF compartments

Chloride (Cl−): (major ECF anion) normal serum level of chloride: 97-107 mEq/L

Functions -Major component of interstitial and lymph fluid; gastric and pancreatic juices, sweat, bile, and saliva -Acts with sodium to maintain the osmotic pressure -Combines with hydrogen ions to produce hydrochloric acid Sources and Losses -Enters body via gastrointestinal tract -Almost all chloride in diet comes from salt -Found in foods high in sodium, processed foods Regulation -Normally paired with sodium; excreted and conserved with sodium by the kidneys -Regulated by aldosterone -Low potassium level leads to low chloride level

Magnesium (Mg2+): (second most abundant ICF cation after potassium) normal serum concentration of magnesium: 1.3-2.3 mEq/L

Functions -Metabolism of carbohydrates and proteins -Role in neuromuscular function -Acts on cardiovascular system, producing vasodilation Sources and Losses -Enters the body via gastrointestinal tract -Sources include green, leafy vegetables; nuts; seafood; whole grains; dried peas and beans; cocoa -Lost via urine with use of loop diuretics Regulation -Eliminated by kidneys -Regulated by parathyroid hormone

Bicarbonate (HCO3−): (anion that is the major chemical base buffer within the body; found in both ECF and ICF) normal serum bicarbonate level: 25-29 mEq/L

Functions -Regulates acid-base balance Sources and Losses -Losses possible via diarrhea, diuretics, and early renal insufficiency -Excess possible via over-ingestion of acid neutralizers, such as sodium bicarbonate Regulation -Bicarbonate levels regulated primarily by the kidneys -Bicarbonate readily available as a result of carbon dioxide formation during metabolism

Phosphate (PO4−): (major ICF anion; a buffer anion in both ICF and ECF) normal serum phosphate level: 2.5-4.5 mg/dL

Functions -Role in acid-base balance as a hydrogen buffer -Promotes energy storage; carbohydrate, protein, and fat metabolism -Bone and teeth formation -Role in muscle and red blood cell function Sources and Losses -Enters body via gastrointestinal tract -Sources include all animal products (meat, poultry, eggs, milk, bread, ready-to-eat cereal) -Absorption is diminished by concurrent ingestion of calcium, magnesium, and aluminum Regulation -Eliminated by kidneys -Regulation by parathyroid hormone and by activated vitamin D -Phosphate and calcium are inversely proportional; an increase in one results in a decrease in the other

Calcium (Ca2+): (most abundant electrolyte in the body) normal total serum calcium level: 8.6-10.2 mg/dL normal ionized serum calcium level: 4.5-5.1 mg/dL

Functions -Role in blood coagulation and in transmission of nerve impulses -Helps regulate muscle contraction and relaxation -Major component of bones and teeth Sources and Losses -Absorbed from foods in the presence of normal gastric acidity and vitamin D -Lost via feces and urine -Sources include milk and milk products; dried beans; green, leafy vegetables; small fish with bones; and dried peas and beans Regulation -Primarily excreted by gastrointestinal tract; lesser extent by kidneys -Regulated by parathyroid hormone and calcitonin -High serum phosphate results in decreased serum calcium; low serum phosphate leads to increased serum calcium

Major Electrolytes: Sodium (Na+): (chief electrolyte of ECF) normal serum concentration of sodium: 135-145 mEq/L

Functions Regulates extracellular fluid volume; Na+ loss or gain accompanied by a loss or gain of water Affects serum osmolality Role in muscle contraction and transmission of nerve impulses Regulation of acid-base balance as sodium bicarbonate Sources and Losses -Normally enters the body through the gastrointestinal tract from dietary sources, such as salt added to processed foods, sodium preservatives added to processed foods -Lost from gastrointestinal tract, kidneys, and skin Regulation -Transported out of the cell by the sodium-potassium pump -Regulated by renin-angiotensin-aldosterone system -Elimination and reabsorption regulated by the kidneys Sodium concentrations affected by salt and water intake

Thyroid gland -Increases blood flow in the body by releasing thyroxine, leading to increased renal circulation and resulting in increased glomerular filtration and urinary output. Nervous system -Inhibits and stimulates mechanisms influencing fluid balance; acts chiefly to regulate sodium and water intake and excretion -Regulates oral intake by sensing intracellular dehydration, which triggers thirst (thirst center located in the hypothalamus) -Neurons, called osmoreceptors, are sensitive to changes in the concentration of ECF, sending appropriate impulses to the pituitary gland to release ADH or inhibit its release to maintain ECF volume concentration.

Gastrointestinal tract -Absorbs water and nutrients that enter the body through this route. Parathyroid glands -Regulate calcium (Ca2+) and phosphate (HPO42−) balance by means of parathyroid hormone (PTH); PTH influences bone reabsorption, calcium absorption from the intestines, and calcium reabsorption from the renal tubules. -Increased secretion of PTH causes elevated serum calcium concentration and lowered serum phosphate concentration. -Decreased secretion of PTH causes lowered serum calcium concentration and elevated serum phosphate concentration.

Keeping track of the shifts That continuous shifting of fluids can have important implications for patient care. For instance, if a hypotonic fluid, such as half-normal saline solution, is given to a patient, it may cause too much fluid to move from the veins into the cells, and the cells can swell. On the other hand, if a hypertonic solution, such as dextrose 5% in normal saline solution, is given to a patient, it may cause too much fluid to be pulled from cells into the bloodstream, and the cells shrink.

Going with the flow In diffusion, solutes move from an area of higher concentration to an area of lower concentration, which eventually results in an equal distribution of solutes within the two areas. Diffusion is a form of passive transport because no energy is required to make it happen; it just happens. Like fish swimming with the current, the solutes simply go with the flow

Maintaining fluid balance Kidneys • Nephrons form urine by filtering blood. • If the body needs more fluid, nephron tubules retain or reabsorb water and electrolytes. • If the body needs less fluid, tubules absorb less, causing more fluids and electrolytes to be excreted. • Kidneys also secrete renin, an enzyme that activates the renin-angiotensin-aldosterone system. • Aldosterone secreted by the adrenal cortex regulates sodium and water reabsorption by the kidneys.

Hormones • ADH—Also known as vasopressin, ADH is produced by the hypothalamus to reduce diuresis and increase water retention if serum osmolality increases or blood volume decreases. • Renin-angiotensin-aldosterone system—If blood flow decreases, the juxtaglomerular cells in the kidneys secrete renin, which leads to the production of angiotensin II, a powerful vasoconstrictor; angiotensin II stimulates the production of aldosterone; aldosterone regulates the reabsorption of sodium and water in the nephron. • ANP—This hormone, produced and stored in the atria of the heart, stops the action of the reninangiotensin-aldosterone system; ANP decreases blood pressure by causing vasodilation and reduces fluid volume by increasing excretion of sodium and water.

Hypotonic: (Get the lowdown) A hypotonic solution has a lower solute concentration than another solution. For instance, say one solution contains only one part sodium and another solution contains two parts. The first solution is hypotonic compared with the second solution. As a result, fluid from the hypotonic solution would shift into the second solution until the two solutions had equal concentrations of sodium. Remember that the body constantly strives to maintain a state of balance, or equilibrium (also known as homeostasis). Understanding hypotonic fluids When a less concentrated, or hypotonic, solution is placed next to a more concentrated solution, fluid shifts from the hypotonic solution into the more concentrated compartment to equalize concentrations. Half-normal saline solution is considered hypotonic because the concentration of sodium in the solution is less than the concentration of sodium in the patient's blood.

Hypertonic: (Just the highlights) A hypertonic solution has a higher solute concentration than another solution. For instance, say one solution contains a large amount of sodium and a second solution contains hardly any. The first solution is hypertonic compared with the second solution. As a result, fluid from the second solution would shift into the hypertonic solution until the two solutions had equal concentrations. Again, the body constantly strives to maintain a state of equilibrium (homeostasis) Understanding hypertonic fluids If one solution has more solutes than an adjacent solution, it has less fluid relative to the adjacent solution. Fluid will move out of the less concentrated solution into the more concentrated, or hypertonic, solution until both solutions have the same amount of solutes and fluid. dextrose 5% in normal saline solution is considered hypertonic because the concentration of solutes in the solution is greater than the concentration of solutes in the patient's blood.

Balancing fluids review Calcium basics • Positively charged ion (cation) • Ninety-nine percent in bones and teeth; 1% in extracellular fluid • Important for bone and tooth formation, normal cell function, and neural transmission • Affects contraction of muscles, blood clotting, and hormone balance • Mostly bound to albumin (always look at albumin level with calcium) • Normal total serum calcium levels: 8.9 to 10.1 mg/dl • Normal ionized calcium levels: adults, 4.4 to 5.3 mg/dl; children, 4.4 to 6.0 mg/dl; elderly, 2.3 to 4.1 mg/dl. Calcium balance • Calcium level is affected by dietary intake and existing stores of calcium in the body. • PTH is released by the parathyroid gland when calcium stores are low; it pulls calcium from bones and promotes transfer of calcium into plasma, kidney reabsorption of calcium, and absorption from intestines. • Calcitonin, another hormone released by the thyroid, antagonizes PTH; if calcium levels are too high, calcitonin inhibits bone resorption, decreases absorption of calcium, and increases excretion of calcium by the kidneys. • Vitamin D promotes calcium absorption from intestines, resorption from bones, and reabsorption by kidneys to increase calcium levels. • Phosphorus is inversely related to calcium and inhibits calcium absorption from intestines; when calcium levels are low, kidneys retain calcium and excrete phosphorus. • Serum pH has an inverse relationship with ionized calcium. If pH rises, more calcium binds with protein and ionized calcium level drops; if pH drops, less calcium binds to protein and ionized calcium level rises.

Hypocalcemia • Major cause: hypoalbuminemia • Also caused by poor dietary intake, malabsorption, pancreatitis, parathyroid and thyroid gland surgery, medications, kidney failure, hypomagnesemia, hyperphosphatemia, and alkalosis Signs and symptoms • Predominantly neuromuscular and cardiovascular • Classic sign: tetany evidenced by Trousseau's and Chvostek's signs • Also include anxiety; confusion; irritability; decreased cardiac output; arrhythmias; prolonged ST segment and QT intervals; fractures; muscle cramps; tremors; twitching; and paresthesia of the face, fingers, and toes Treatment • I.V. calcium gluconate or calcium chloride • Possibly magnesium supplements, vitamin D supplements, and adequate dietary intake of calcium (for chronic hypocalcemia)

Hypervolemia • Most common fluid imbalance associated with heart failure • Results from heart's failure to propel blood forward, resulting in vascular pooling and sodium and water reabsorption • Commonly causes peripheral edema Hypovolemia • Associated with overuse of diuretics • Causes confusion and hypotension in elderly patients • May cause electrolyte imbalances

Hyponatremia • May result from sodium loss due to diuretic abuse • May result from a dilutional effect when water reabsorption is greater than sodium reabsorption • May cause confusion Other electrolyte imbalances • Hypokalemia—results from prolonged use of a diuretic without adequate potassium replacement • Hyperkalemia—occurs with use of potassium-sparing diuretics • Hypokalemia and hyperkalemia—lead to life-threatening arrhythmias • Hypomagnesemia—occurs with hypokalemia, especially with diuretic use • Hypochloremia—results from excessive diuretic therapy

When you're trying to think of the four most common complications of I.V. therapy, remember that getting any complication is a PITI: Phlebitis Infiltration Thrombophlebitis Infection I.V. fluid replacement review Types of I.V. solutions • Broadly classified as crystalloids or colloids Crystalloids • Solutions with small molecules that flow easily from the bloodstream into cells and tissues • May be isotonic, hypotonic, or hypertonic Isotonic solutions • Contain about the same concentration of osmotically active particles as extracellular fluid, so fluid doesn't shift between extracellular and intracellular spaces • Osmolality: 240 to 340 mOsm/kg • Example: D5W, normal saline solution, and dextrose 5% in normal saline solution

Hypotonic solutions • Are less concentrated than extracellular fluid, which allows movement from the bloodstream into the cells, causing cells to expand • Osmolality: less than 240 mOsm/kg • Example: half-normal saline solution • Can cause cardiovascular collapse from vascular fluid depletion or increased ICP from fluid shifting into brain cells • Avoid using in patients at risk for increased ICP, such as those who have had a stroke, head trauma, or neurosurgery • Also, avoid using in patients who suffer from abnormal fluid shifts into the interstitial space or body cavities, such as in liver disease, burns, or trauma Hypertonic solutions • Are more concentrated than extracellular fluid, which allows movement of fluid from cells into the bloodstream, causing cells to shrink • Osmolality greater than 340 mOsm/kg • Examples include dextrose 5% in half-normal saline solution, 3% sodium chloride solution, and dextrose 10% in normal saline solution • May not be tolerated by those with cardiac or renal disease • May cause fluid overload and pulmonary edema • Should not be used in patients at risk for cellular dehydration, such as those with DKA Colloids • Act as plasma expanders • Are always hypertonic, pulling fluid from cells into the bloodstream • Examples: albumin, plasma protein fraction, dextran, and hetastarch • Require close monitoring for signs and symptoms of hypervolemia, such as increased blood pressure, dyspnea, and bounding pulse

Respiratory failure review Respiratory failure basics • Occurs when the lungs can't sufficiently maintain arterial oxygenation or eliminate carbon dioxide • Usually produces hypercapnia and hypoxemia in patients with normal lung tissue • May be signaled only by an acute drop in ABG levels and clinical deterioration in patients with COPD • Gas exchange diminished by three major malfunctions: alveolar hypoventilation, (V/Q) mismatch, and intrapulmonary shunting

Imbalances associated with respiratory failure Hypervolemia • May be caused by excessive fluid absorption as a result of prolonged respiratory treatments or increased lung capillary pressure or permeability • May precipitate pulmonary edema Hypovolemia • May be caused by increased respiratory rate, which can promote excessive water loss • May also occur with fever or any other condition that increases the metabolic rate and thus the respiratory rate Hypokalemia • May be caused by hyperventilation and resulting alkalosis, which causes hydrogen ions to move out of the cells and potassium ions to shift from the blood and into the cells Hyperkalemia • May be caused by acidosis, in which excess hydrogen ions move into the cells while potassium ions shift from the cells and into the blood

Acute pancreatitis review Acute pancreatitis basics • Inflammation of the pancreas • Two types: edematous, which is usually mild, accounts for about 85% of cases, is self-limiting, and resolves in 5 to 7 days; and necrotizing, which is severe, accounts for about 15% of cases, is progressive, and causes tissue damage and cell death • May progress to chronic pancreatitis with progressive recurrent episodes Causes • Gallstones most common cause • Alcohol consumption second most common cause • Other biliary tract disease, drugs, infection, toxins and metabolic processes, trauma, and other factors less common causes Effects on pancreas • Exocrine functions fail • Activated enzymes in pancreas digest pancreatic tissue

Imbalances caused by acute pancreatitis Hypovolemia • Major cause of death in acute pancreatitis • Occurs when severe pancreatic damage triggers release of systemic inflammatory mediators that produce increased capillary permeability and vasodilation, leading to massive fluid shifts from intravascular to interstitial spaces and retroperitoneum • Can also result from vomiting, diarrhea, excessive sweating, and, possibly, hemorrhage Hyponatremia • Can result from vomiting, diarrhea, and excessive sweating • Also occurs when hypovolemia causes an increase in antidiuretic hormone secretion Hypocalcemia • Usually results from concomitant hypoalbuminemia • Can be worsened by fat necrosis (caused by lipase necrosing fat tissue in pancreatic interstitium and peripancreatic spaces) that may result in the release of free fatty acids and intraperitoneal saponification • Can also stem from hypomagnesemia (hypomagnesemia should be addressed before hypocalcemia)

Renal failure review Renal failure basics • Involves disruption of normal kidney function • Affects the kidney's functional unit, the nephron, which forms urine • Results in kidneys losing the ability to excrete water, electrolytes, wastes, and acid-base products through the urine, causing imbalances • May lead to development of hypertension, anemia, uremia, and renal osteodystrophy • May be acute or chronic Acute renal failure • Occurs suddenly • Usually reversible • May stem from prerenal, intrarenal, or obstructive postrenal conditions • Divided into three phases: oliguric-anuric, diuretic, and recovery Chronic renal failure • Occurs slowly • Irreversible • May stem from chronic glomerular disease, chronic infections, congenital anomalies, vascular disease, long-term therapy with nephrotoxic drugs, and endocrine diseases • Divided into four stages: reduced renal reserve, renal insufficiency, renal failure, and end-stage renal disease

Imbalances caused by renal failure Hypervolemia • Occurs when urine output decreases and the body retains fluid or when fluid intake exceeds urine output • May lead to hypertension, peripheral edema, heart failure, or pulmonary edema Hypovolemia • Usually occurs during the diuretic phase of acute renal failure • May result in hypotension or circulatory collapse Hyperkalemia • Occurs as the kidneys' ability to excrete potassium is impaired • May also occur because metabolic acidosis, which occurs with renal failure, causes potassium to move from inside the cells into the extracellular fluid • May be exacerbated by release of potassium from necrotic or injured cells • Can also be caused by stressors, such as infection, GI bleeding, trauma, and surgery

Documenting electrolyte imbalances Be sure to include the following information in your documentation of a patient's electrolyte imbalance: • assessment findings • laboratory results pertaining to the imbalance • related nursing diagnoses • notification and response of the practitioner • interventions and treatment for the electrolyte imbalance, including safety measures • patient teaching • patient's response to interventions.

Improving your I.V. IQ To evaluate I.V. fluid treatment, ask: • Is the I.V. fluid providing the correct amount of electrolytes? • How long has the patient been receiving I.V. fluids? • Is the patient receiving oral supplementation of electrolytes?

Infants have considerably more total body fluid and ECF than adults. Because ECF is more easily lost from the body than ICF, infants are at increased risk for fluid volume deficits. Total body water also differs by: -biological sex -amount of fat cells in the body. Fat cells contain little water, whereas lean tissue is rich in water. -more obese a person is, the smaller the person's percentage of total body water is when compared with body weight. Because women tend to have proportionally more body fat than men do, they also have less body fluid than men. -decreasing percentage of body fluid in older people is related to an increase in fat cells. -older adults lose muscle mass as a part of aging. The combined increase of fat and loss of muscle results in reduced total body water; after the age of 60, total body water is about 45% of a person's body weight. This decrease in water increases the risk for fluid imbalance in older adults.

In general, fluid intake averages 2,600 mL per day, with approximately 1,300 mL coming from ingested water, 1,000 mL coming from ingested food, and 300 mL from metabolic oxidation.

Fluid types Three types of solutions: -isotonic -hypotonic -hypertonic

Isotonic: (Already at match point) An isotonic solution has the same solute (matter dissolved in solution) concentration as another solution. No net fluid shifts occur between isotonic solutions because the solutions are equally concentrated Normal saline solution is considered isotonic because the concentration of sodium in the solution nearly equals the concentration of sodium in the blood.

The effect of diuretics • Treat hypertension, heart failure, electrolyte imbalances, and kidney disease • Increase urine production • Cause loss of electrolytes, particularly potassium • Require careful monitoring of electrolytes

Key issues in I.V. fluid treatment • Patient's normal electrolyte requirements • Correct amount of electrolytes prescribed and given • Length of treatment • Concomitant oral electrolyte supplementation

Organs and Body Systems Related to the Regulation of Fluid and Electrolyte Balance

Kidneys -Regulate extracellular fluid (ECF) volume and osmolality by selective retention and excretion of body fluids -Regulate electrolyte levels in the ECF by selective retention of needed substances and excretion of unneeded substances -Regulate pH of ECF by excretion or retention of hydrogen ions -Excrete metabolic wastes (primarily acids) and toxic substances -Normally filter 180 L of plasma daily in the adult, while excreting only 1.5 L of urine

Giving that extra push In active transport, solutes move from an area of lower concentration to an area of higher concentration. Like swimming against the current, active transport requires energy to make it happen. The energy required for a solute to move against a concentration gradient comes from a substance called adenosine triphosphate or ATP. Stored in all cells, ATP supplies energy for solute movement in and out of cells

Letting fluids through Osmosis refers to the passive movement of fluid across a membrane from an area of lower solute concentration and comparatively more fluid into an area of higher solute concentration and comparatively less fluid. Osmosis stops when enough fluid has moved through the membrane to equalize the solute concentration on both sides of the membrane.

Heart and blood vessels -Circulate nutrients and water throughout the body -Circulate blood through the kidneys under sufficient pressure for urine to form (pumping action of the heart) -React to hypovolemia by stimulating fluid retention (stretch receptors in the atria and blood vessels)

Lungs -Remove approximately 300 mL of water daily through exhalation (insensible water loss) in the normal adult -Eliminate about 13,000 mEq of hydrogen ions (H+) daily, as opposed to only 40 to 80 mEq excreted daily by the kidneys -Act promptly to correct metabolic acid-base disturbances; regulate H+ concentration (pH) by controlling the level of carbon dioxide (CO2) in the extracellular fluid

Magnesium imbalances review Magnesium basics • Second most abundant cation in intracellular fluid • Performs many functions - Promotes enzyme reactions within the cells during carbohydrate metabolism - Helps the body produce and use ATP for energy - Takes part in DNA and protein synthesis - Influences vasodilation and cardiac muscle contractility - Aids in neurotransmission - Plays an essential role in the production of parathyroid hormone - Helps sodium and potassium cross cell membranes • Normal serum levels: 1.5 to 2.5 mEq/L; ranges differ for neonates and children

Magnesium balance • More than half of magnesium ions are free, circulating ions; others bind to albumin and other substances. • Magnesium levels relate to albumin levels; low magnesium equals low albumin; high magnesium equals high albumin. • GI and urinary systems regulate magnesium levels to maintain balance.

positive charge ions= cations Major cations (positive charge) in body fluid: -sodium -potassium -calcium -hydrogen -magnesium negative charge ions= anions Major anions ( negative charge) in body fluid: -chloride -bicarbonate -phosphate without a charge= nonelectrolytes -urea -glucose

Major electrolytes in the ECF include sodium, chloride, calcium, and bicarbonate. Major electrolytes in the ICF include potassium, phosphorus, and magnesium. To remind yourself about the difference between anions and cations, remember that the T in "cation" looks like the positive symbol, "+."

Balancing electrolytes review Electrolyte basics • Found throughout the body in various concentrations • Critical to cell function Ions, anions, cations • Ions—electrically charged particles created when electrolytes separate in a solution; may be positively or negatively charged • Anions—negatively charged electrolytes; include chloride, phosphorus, and bicarbonate • Cations—positively charged electrolytes; include sodium, potassium, calcium, and magnesium • Electroneutrality—positive and negative ions balance each other out, achieving a neutral electrical charge

Major extracellular electrolytes • Sodium—helps nerve cells and muscle cells interact • Chloride—maintains osmotic pressure and helps gastric mucosal cells produce hydrochloric acid • Calcium—stabilizes cell membrane, reducing its permeability; transmits nerve impulses; contracts muscles; coagulates blood; and forms bones and teeth • Bicarbonate—regulates acid-base balance Major intracellular electrolytes • Potassium—regulates cell excitability, nerve impulse conduction, resting membrane potential, muscle contraction, myocardial membrane responsiveness, and intracellular osmolality • Phosphate—controls energy metabolism • Magnesium—influences enzyme reactions, neuromuscular contractions, normal functioning of nervous and cardiovascular system, protein synthesis, and sodium and potassium ion transportation

Imbalances associated with excessive GI fluid loss Hypovolemia and dehydration • Can occur with prolonged vomiting and diarrhea or if gastric and intestinal suctioning occur without proper monitoring of intake and output Hypokalemia • Can occur with excessive loss of gastric fluids rich in potassium Hypomagnesemia • Can occur with prolonged (lasting several weeks) vomiting, diarrhea, or gastric suctioning Hyponatremia • Can occur with prolonged vomiting, diarrhea, or gastric suctioning • Can also occur with excessive use of tap water enemas because water absorbed by the colon can have a dilutional effect on sodium Hypochloremia • Can be caused by any loss of gastric contents

Metabolic acidosis • Caused by a loss of intestinal fluid, which causes loss of bicarbonate, leading to decreased pH and an acidic state Metabolic alkalosis • Caused by a loss of gastric fluids, including acid, which increases pH and creates an alkalotic state Hypermagnesemia and hyperphosphatemia • May be caused by excessive use of laxatives, such as magnesium sulfate, milk of magnesia, and Fleet Phospho-soda Hypernatremia and hyperphosphatemia • May be caused by excessive use of enemas containing sodium and phosphorus, such as Fleet enemas Treatment • Prevention of further fluid and electrolyte loss • Antiemetics for nausea and vomiting if indicated • Antidiarrheals if diarrhea is the cause • I.V. or oral fluid replacement, depending on the severity • Electrolyte replacement • Long-term parenteral nutrition if needed • Antibiotics if infection is the underlying cause

Hyperphosphatemia • Develops when the kidneys lose the ability to excrete phosphorus Hypocalcemia • Occurs when phosphorus levels increase (calcium and phosphorus have an inverse relationship) • May also occur because decreased activation of vitamin D by the kidneys results in decreased GI absorption of calcium Hyponatremia • Occurs with acute renal failure because decreased GFR and damaged tubules increase water and sodium retention • Can also be caused by the intracellular-extracellular exchange between sodium and potassium during metabolic acidosis Hypernatremia • Can occur with chronic kidney disease because progressive kidney failure results in the excretion of less sodium Hypermagnesemia • May result from decreased GFR and destruction of tubules • Isn't usually apparent unless the patient is receiving external sources of magnesium, such as laxatives, antacids, I.V. solutions, or hyperalimentation solutions

Metabolic acidosis • Is the most common acid-base imbalance occurring with renal failure • Occurs when the kidneys lose their ability to secrete hydrogen ions (acid) in the urine • Also occurs when the kidneys fail to store bicarbonate (base) Metabolic alkalosis • Rarely results from excessive intake of bicarbonate, which may be given to correct metabolic acidosis Treatment • Correction of specific symptoms • Treatment of the underlying disease • Low-protein, low-sodium, low-potassium, high-calorie diet • Maintenance of fluid and electrolyte balance • Erythropoietin to stimulate production of RBCs in the bone marrow • Possibly hemodialysis or peritoneal dialysis • Possibly a kidney transplant

Metabolic acidosis • Occurs when hydrogen production increases • Depresses the CNS and, if untreated, may lead to ventricular arrhythmias, coma, and cardiac arrest Causes • Loss of bicarbonate (base) • Accumulation of metabolic acids (acid) • Overproduction of ketone bodies • Decreased ability of kidneys to excrete acids • Excessive GI losses from diarrhea, intestinal malabsorption, or urinary diversion to ileum • Hyperaldosteronism • Use of potassium-sparing diuretics • Poisoning or toxic drug reaction Treatment • Correction of acidosis as quickly as possible • Respiratory compensation (mechanical ventilation if needed) • Rapid-acting insulin (for diabetics) • I.V. bicarbonate and I.V. fluids • Dialysis (for patients with renal failure)

Metabolic alkalosis • Commonly associated with hypokalemia • Results from decrease in hydrogen production, a gain in bicarbonate, or both Causes • Excessive acid loss from the GI tract • Diuretic therapy (kidney loss of hydrogen, potassium, and chloride) • Cushing's disease (from sodium and chloride retention and potassium and hydrogen excretion) Treatment • I.V. fluids and acetazolamide to increase renal excretion • Correction of underlying acid-base imbalance • Discontinuation of thiazide diuretics and NG suctioning For severe cases • I.V. ammonia chloride

Common TPN additives Electrolytes • Calcium: promotes development and maintenance of bones and teeth and aids in blood clotting • Chloride: regulates acid-base balance and maintains osmotic pressure • Magnesium: helps the body absorb carbohydrates and protein • Phosphorus: essential for cell energy and calcium balance • Sodium: helps control water distribution and maintains normal fluid balance. Vitamins • Folic acid: helps with DNA formation and promotes growth and development • Vitamin B complex: helps the final absorption of carbohydrates and protein • Vitamin C: helps in wound healing • Vitamin D: essential for bone metabolism and maintenance of serum calcium levels • Vitamin K: helps prevent bleeding disorders

Other additives • Micronutrients (zinc, copper, chromium, selenium, manganese): help in wound healing and RBC synthesis • Amino acids: provide the proteins necessary for tissue repair and immune functions • Lipids: support hormone and prostaglandin synthesis; prevent essential fatty acid deficiency Lipid emulsions • Thick preparations that supply patients with essential fatty acids and calories • Assist in wound healing, RBC production, and prostaglandin synthesis • May be piggybacked with TPN • Should be used cautiously in patients with hepatic or pulmonary disease, acute pancreatitis, anemia, or a coagulation disorder and in patients at risk for developing a fat embolism • Should be avoided in patients with pathologic hyperlipidemia or lipid nephrosis

To remember some of the signs and symptoms of hyperphosphatemia, think of the word CHEMO (keeping in mind that chemotherapy can lead to hyperphosphatemia): Cardiac irregularities Hyperreflexia Eating poorly Muscle weakness Oliguria.

Phosphorus imbalances review Phosphorus basics • Primary anion in intracellular fluid • About 85% found in bones and teeth, combined with calcium in a 1:2 ratio • Crucial to cell membrane integrity, muscle and neurologic function, and metabolism of carbohydrates, fats, and proteins -Promotes oxygen delivery from RBCs to tissues • Buffers acids and bases, promotes energy transfer by forming ATP, and is essential for healthy bones and teeth • Normal range: 2.5 to 4.5 mg/dl (1.8 to 2.6 mEq/L) Phosphorus balance • Dietary intake and renal excretion maintain normal levels; if intake increases, renal excretion also increases. • The parathyroid gland controls hormonal regulation of phosphorus levels by affecting PTH. • PTH release is affected by calcium level; PTH causes the kidneys to increase excretion of phosphorus if the calcium level is high and to reabsorb phosphorus if the calcium level is low. • Balance is affected by certain conditions that cause transcellular shift of phosphorus; for example, insulin moves glucose and phosphorus into cells; alkalosis causes the same kind of shift.

Adrenal glands -Regulate blood volume and sodium and potassium balance by secreting aldosterone, a mineral corticoid secreted by the adrenal cortex, causing sodium retention (and thus water retention) and potassium loss. -Decreased secretion of aldosterone causes sodium and water loss and potassium retention. -Cortisol, another adrenocortical hormone, has only a fraction of the potency of aldosterone. However, secretion of cortisol in large quantities can produce sodium and water retention and potassium deficit.

Pituitary gland -Stores and releases the antidiuretic hormone (ADH) (manufactured in the hypothalamus), which acts to allow the body to retain water. It acts chiefly to regulate sodium and water intake and excretion. -When osmotic pressure of the ECF is greater than that of the cells (as in hypernatremia—excess sodium—or hyperglycemia), ADH secretion is increased, causing renal retention of water. -When osmotic pressure of the ECF is less than that of the cells (as in hyponatremia), ADH secretion is decreased, causing renal excretion of water. -When blood volume is decreased, an increased secretion of ADH results in water conservation. When blood volume is increased, a decreased secretion of ADH results in water loss.

Antidiuretic hormone Several hormones affect fluid balance, among them a water retainer called antidiuretic hormone (ADH). (You may also hear this hormone called vasopressin.) The hypothalamus produces ADH, but the posterior pituitary gland stores and releases it.

Remember what ADH stands for—antiduretic hormone—and you'll remember its job: restoring blood volume by reducing diuresis and increasing water retention.

Respiratory acidosis • Results from compromise in breathing • Characterized by alveolar hypoventilation (body can't get rid of carbon dioxide) • Leads to hypercapnia Causes • Hypoventilation from CNS trauma or tumor that depresses the respiratory center • Neuromuscular disorders that affect respiratory drive • Lung diseases that decrease amount of surface area available for gas exchange • Airway obstruction • Chest wall trauma • Drugs that depress the respiratory center Treatment • Ventilation, bronchodilator, supplemental oxygen, and chest physiotherapy • Antibiotics to treat infection • Drug therapy to treat hyperkalemia • Removal of foreign bodies from airway if needed • Pain management

Respiratory alkalosis • Occurs when carbon dioxide elimination increases Causes • Conditions that increase respiratory rate and depth • Hyperventilation • Hypercapnia • Hypermetabolic states • Liver failure • Certain drugs • Conditions that affect the brain's respiratory control center • Acute hypoxia secondary to high altitude, pulmonary disease, severe anemia, pulmonary embolus, and hypotension Treatment • Removal of causative agent • Fever reduction • Sepsis treatment • Oxygen therapy (if acute hypoxemia is the cause) • Rebreathing exhaled carbon dioxide

Maintaining acid-base balance Three systems regulate acids and bases: • Chemical buffers—neutralize the offending acid or base • Respiratory system—regulates retention and excretion of acids • Kidneys—excrete or retain acids or bases Chemical buffer systems • Bicarbonate buffer system—buffers blood and interstitial fluid • Phosphate buffer system—reacts with acids and bases to form compounds that alter pH; especially effective in the renal tubules • Protein buffer system—acts inside and outside the cell; binds with acids and bases to neutralize them.

Respiratory system • Functions as the second line of defense • Responds to pH changes in minutes • Makes temporary adjustments to pH • Regulates carbon dioxide levels in the blood by varying the rate and depth of breathing • Compensates with quick and deep breathing so more carbon dioxide is lost when bicarbonate levels are low • Compensates with slow, shallow breathing so more carbon dioxide is retained when bicarbonate levels are high • Regulates carbonic acid production Kidneys • Kick in when the first two systems fail to reverse the acidosis or alkalosis • Make long-term adjustments to pH • Reabsorb acids and bases or excrete them into urine • Produce bicarbonate to replenish lost supply • Regulate bicarbonate production • Compensate with bicarbonate retention and increased acid excretion when PaCO2 level is high • Respond with bicarbonate excretion and increased acid retention when PaCO2 level is low

Hyponatremia • Common electrolyte imbalance • Caused by an inadequate sodium intake, excessive water loss, or water gain • Serum sodium level less than 135 mEq/L • Varied signs and symptoms, depending on the individual • Results from decreased serum osmolality • Fluid shifts into intracellular areas: neurologic symptoms are related to cerebral edema • May cause stupor and coma if serum sodium level drops to 110 mEq/L Types • Hypovolemic—both sodium and water are decreased in extracellular area, but sodium loss is greater than water loss • Hypervolemic—both sodium and water are increased in extracellular area, but water gain is more than sodium gain • Isovolemic—water increases, but total sodium levels remain stable; may also be caused by SIADH

Signs and symptoms • Abdominal cramps • Lethargy and confusion (altered LOC) • Headache • Muscle twitching • Nausea and vomiting • Anorexia Signs and symptoms with depletional hyponatremia • Dry mucous membranes • Orthostatic hypotension • Poor skin turgor • Tachycardia Signs and symptoms with dilutional hyponatremia • Hypertension • Rapid, bounding pulse • Weight gain

Hypomagnesemia • Occurs when serum magnesium levels are less than 1.5 mEq/L • Results from poor dietary intake of magnesium, poor GI absorption, and increased loss from GI tract or urinary tract • Occurs in patients who are pregnant; those with chronic diarrhea, hemodialysis, hypercalcemia, hypothermia, sepsis, burns, and wound debridement; and patients taking certain medications

Signs and symptoms • Altered LOC • Ataxia • Confusion • Depression • Hallucinations and/or delusions • Seizures • Vertigo • Skeletal muscle weakness • Hyperactive DTRs • Tetany • Chvostek's and Trousseau's signs • Arrhythmias • Rapid heart rate • Vomiting • Insomnia Treatment • Change in diet • Oral or I.V. magnesium replacement

Excessive GI fluid loss review Excessive GI fluid loss • May result from vomiting, suctioning, or increased or decreased GI tract motility • May be excreted as waste products or secreted from the intestinal wall into the intestinal lumen, leading to fluid and electrolyte imbalances Causes • Anorexia nervosa • Antibiotic use • Bacterial GI infections • Bulimia • Enteral tube feedings and ostomies • Excessive intake of alcohol and illicit drugs • Excessive use of enemas and laxatives • Fecal impaction • GI bleeding • GI fistulas • Intestinal obstruction • Pancreatitis or hepatitis • Paralytic ileus • Poor absorption of foods • Poor digestion • Pyloric stenosis in young children

Signs and symptoms • Altered respirations • Arrhythmias • Cool, dry skin or decreased skin turgor • Decreased, concentrated urine output • Falsely elevated hemoglobin level and hematocrit • Increased heart rate and decreased blood pressure • Sunken eyeballs • Muscle cramps • Weakness and confusion

Hypermagnesemia • Occurs when serum magnesium levels are greater than 2.5 mEq/L • Is usually uncommon except in patients with renal failure (especially patients taking antacids or laxatives or the elderly with decreased renal function) • Caused by Addison's disease, adrenocortical insufficiency, and untreated DKA • May result from increased intake of magnesium, usually from hemodialysis using magnesium-rich dialysate, TPN with excess magnesium, or continuous magnesium sulfate infusion to treat certain conditions

Signs and symptoms • Decreased muscle and nerve activity • Hypoactive DTRs • Generalized weakness, drowsiness, and lethargy • Facial paresthesias • Nausea and vomiting • Slow, shallow, depressed respirations or respiratory paralysis • Respiratory arrest • ECG changes • Vasodilation and hypotension • Arrhythmias and bradycardia Treatment • Oral or I.V. fluids • Avoidance of magnesium products • Calcium gluconate, in emergent situations • Hemodialysis with magnesium-free dialysate (for dialysis patients) • Mechanical ventilation (for severe cases in which respiration depression is present)

Hypercalcemia • Common electrolyte disorder • Considered a metabolic emergency • Two major causes: primary hyperparathyroidism, which releases excess PTH, and cancer, which releases a substance similar to PTH • Other causes: hyperthyroidism, fractures, prolonged immobilization, hypophosphatemia, acidosis, vitamin A overdose, and certain medications

Signs and symptoms • Heart, skeletal muscle, and nervous system are most affected. • Include confusion, lethargy, depression, altered mental status, muscle weakness, hyporeflexia, characteristic ECG changes, hypertension, bone pain, abdominal pain and constipation, nausea, vomiting, anorexia, polyuria, and extreme thirst Treatment • Hydration • Decreased calcium intake • Diuretics, corticosteroids, bisphosphonates, and plicamycin • Hemodialysis or peritoneal dialysis (for life-threatening cases)

Respiratory acidosis • Results from hypoventilation (lungs can't eliminate carbon dioxide; carbon dioxide combines with water to form carbonic acid) • Contributed to by decreased pH levels that result from increased carbonic acid Respiratory alkalosis • Results from hyperventilation (rapid respiration causes excessive carbon dioxide elimination, which decreases the blood's acid-forming potential, resulting in alkalosis) Metabolic acidosis • Results from hypoxia, which causes cells to use anaerobic metabolism • Produces an increase in lactic acid, leading to metabolic acidosis

Signs and symptoms • Increased respiratory depth and rate • Muscle retraction between the ribs, above the clavicle, and above the sternum • Increased heart rate and arrhythmias • Constriction of blood vessels • Anxiety and restlessness, progressing to fatigue, confusion, agitation, and lethargy • Headaches • Changes in serum potassium levels Treatment • Lowest possible dose of oxygen for the shortest amount of time to prevent oxygen toxicity • Intubation and ventilation if unable to achieve oxygen saturation above 90% • Bronchodilators as ordered to open airways • Corticosteroids, theophylline, antibiotics, chest physiotherapy, and suctioning as ordered • I.V. fluids for dehydration or diuretics for a fluid overload as ordered

Hypokalemia • Serum potassium levels less than 3.5 mEq/L(moderate hypokalemia: 2.5 to 3 mEq/L; severe hypokalemia: < 2.5 mEq/L) • Underlying mechanisms: medications or inadequate intake or excessive output of potassium • Caused by prolonged intestinal suction, prolonged vomiting or diarrhea, laxative abuse, severe diaphoresis, recent ileostomy, and villous adenoma

Signs and symptoms • Skeletal muscle weakness, U wave (ECG changes), Constipation, Toxicity (digoxin), Irregular and weak pulse, Orthostatic hypotension, Numbness (SUCTION is the acronym to remember.) • Other signs and symptoms - Anorexia - Cramps - Decreased bowel sounds - ECG changes - Hyporeflexia - Nausea - Paresthesia - Polyuria - Vomiting - Leg cramps - Decreased or absent deep tendon reflexes - Paralysis Treatment • High-potassium diet • Oral potassium supplements • I.V. potassium therapy • Potassium-sparing diuretic, if needed

Hypernatremia • Caused by water loss, inadequate water intake (rarely from failure of the thirst mechanism), excessive sodium intake, or diabetes insipidus • Patients at increased risk: infants, elderly, immobile and comatose patients • Always results in increased serum osmolality • Fluid shifts out of the cells, causing cells to shrink • Must be corrected slowly to prevent a rapid shift of water back into the cells, which could cause cerebral edema Signs and symptoms • Agitation • Confusion • Flushed skin • Lethargy • Low-grade fever • Thirst • Restlessness • Muscle twitching • Weakness

Signs and symptoms of hypervolemia with sodium gain • Bounding pulse • Dyspnea • Hypertension Signs and symptoms of hypervolemia with water loss • Dry mucous membranes • Oliguria • Orthostatic hypotension

Renin-angiotensin-aldosterone system To help the body maintain a balance of sodium and water as well as a healthy blood volume and blood pressure, special cells (called juxtaglomerular cells) near each glomerulus secrete an enzyme called renin. Through a complex series of steps, renin leads to the production of angiotensin II, a powerful vasoconstrictor. Angiotensin II causes peripheral vasoconstriction and stimulates the production of aldosterone. Both actions raise blood pressure

Sodium and water regulator The hormone aldosterone also plays a role in maintaining blood pressure and fluid balance. Secreted by the adrenal cortex, aldosterone regulates the reabsorption of sodium and water within the nephron

Albumin, a large protein molecule, acts like a magnet to attract water and hold it inside the blood vessel.

The kidneys play a vital role in fluid balance. -Nephron consists of a glomerulus and a tubule. The tubule, sometimes convoluted, ends in a collecting duct. The glomerulus is a cluster of capillaries that filters blood. Like a vascular cradle, Bowman's capsule surrounds the glomerulus. Capillary blood pressure forces fluid through the capillary walls and into Bowman's capsule at the proximal end of the tubule. Along the length of the tubule, water and electrolytes are either excreted or retained depending on the body's needs. If the body needs more fluid, for instance, it retains more. If it needs less fluid, less is reabsorbed and more is excreted. Electrolytes, such as sodium and potassium, are either filtered or reabsorbed throughout the same area. The resulting filtrate, which eventually becomes urine, flows through the tubule into the collecting ducts and eventually into the bladder as urine.

Influences on electrolyte balance • Normal cell function • Fluid intake and output • Acid-base balance • Hormone secretion Maintaining electrolyte balance • Most major organs and glands in the body help regulate fluid and electrolyte balance.

The role of organs and glands • Kidneys—regulate sodium and potassium balance (excrete potassium in exchange for sodium retention) • Lungs and liver—regulate sodium and water balance and blood pressure • Heart—secretes ANP, causing sodium excretion • Sweat glands—excrete sodium, potassium, chloride, and water in sweat • GI tract—absorbs and excretes fluids and electrolytes • Parathyroid glands—secrete parathyroid hormone, which draws calcium into the blood and helps move phosphorous to the kidneys for excretion • Thyroid gland—secretes calcitonin, which prevents calcium release from the bone • Hypothalamus and posterior pituitary—produce and secrete antidiuretic hormone causing water retention, which affects solute concentration • Adrenal glands—secrete aldosterone, which influences sodium and potassium balance in the kidneys

Primary body fluid is water, -most important nutrient of life. Water in the body functions primarily to: -Transport nutrients to cells and wastes from cells -Transport hormones, enzymes, blood platelets, and red and white blood cells -Facilitate cellular metabolism and proper cellular chemical functioning -Act as a solvent for electrolytes and nonelectrolytes -Help maintain normal body temperature -Facilitate digestion and promote elimination -Act as a tissue lubricant

Total body water or fluid is the total amount of water, - 50% to 60% of body weight( healthy person)

To remember how to avoid the complication of refeeding syndrome when giving TPN to a severely malnourished patient, think "Start low and go slow.

Total parenteral nutrition review Total parenteral nutrition • Highly concentrated, hypertonic nutrient solution used for patients with high caloric and nutritional needs due to illness or injury • Provides crucial calories; restores nitrogen balance; and replaces essential fluids, vitamins, electrolytes, minerals, and trace elements • Promotes tissue and wound healing and normal metabolic function; gives the bowel a chance to heal; reduces activity in the gallbladder, pancreas, and small intestine; and improves a patient's response to surgery • Used in patients who can't meet their nutritional needs by oral or enteral feedings, including those with inflammatory bowel disease, ulcerative colitis, bowel obstruction or resection, radiation enteritis, severe diarrhea or vomiting, AIDS, chemotherapy, and severe pancreatitis • Typically has limited value in well-nourished patients with GI tracts that are healthy or are likely to resume normal function within 10 days • Must be infused through a central vein

Hyperphosphatemia • Severe: 6 mg/dl or higher • Usually caused by impaired renal excretion of phosphorus • Other causes: hypoparathyroidism (usually after thyroid or parathyroid surgery), resulting in reduced PTH levels and reduced phosphate excretion • Conditions causing shift of phosphorus into extracellular fluid: respiratory acidosis, DKA, cell destruction caused by chemotherapy, necrosis, rhabdomyolysis, trauma, heat stroke, and infection • Also caused by overadministration of phosphorus supplements or phosphorus-containing laxatives and enemas and excessive intake of vitamin D Signs and symptoms • Usually caused by hypocalcemia • Include paresthesia, muscle cramps, muscle weakness, tetany, positive Trousseau's and Chvostek's signs, decreased mental status, hyperreflexia, anorexia, nausea and vomiting, and calcification (which causes arrhythmias, irregular heart rate, decreased urine output, conjunctivitis, cataracts, impaired vision, and papular eruptions)

Treatment • Aimed at correcting underlying problem • Includes low-phosphorus diet and drugs to decrease absorption of phosphorus (aluminum, calcium salts, magnesium [except in those with renal failure], or phosphate-binding antacids) For severe hyperphosphatemia • I.V. saline solution • Proximal diuretics to promote excretion • Dialysis if necessary

Hyperchloremia • Rarely occurs on its own; often associated with other acid-base imbalances (such as metabolic acidosis) • Chloride and sodium closely related; hypernatremia may cause hyperchloremia • Bicarbonate and chloride inversely related; hyperchloremia may occur if bicarbonate decreases • Also may result from increased chloride and decreased water intake, decreased absorption of chloride from intestines, and certain medications Signs and symptoms • Associated with metabolic acidosis (which rarely produces signs and symptoms on its own), such as tachypnea, lethargy, changes in cognition, and weakness Severe metabolic acidosis • Arrhythmias • Kussmaul's respirations • Decreased cardiac output • Decreased LOC that may progress to coma

Treatment • I.V. fluids to speed renal excretion of chloride • Restricted sodium and chloride intake • I.V. sodium bicarbonate for severe hyperchloremia

Hypochloremia • Can be caused by: - poor chloride intake because of a salt-restricted diet, chloride-deficient infant formula, or I.V. fluid replacement without electrolyte supplementation - excessive losses from the GI tract, skin, or kidneys - sodium or potassium deficiency or metabolic alkalosis, diabetic ketoacidosis, Addison's disease, diuretics, rapid removal of ascitic fluid, and heart failure Signs and symptoms • Hyperactive deep tendon reflexes • Muscle hypertonicity and cramps • Signs and symptoms of acid-base imbalance (alkalosis) and electrolyte imbalances (hyponatremia and hypokalemia) • Tetany

Treatment • Increased dietary intake • Treatment of underlying cause of metabolic alkalosis • I.V. saline solution with either sodium chloride or potassium chloride

Hypomagnesemia • Can result from vomiting and diarrhea • Can occur when magnesium is deposited in areas of fat necrosis, decreasing serum levels Hypokalemia • May be caused by severe vomiting and diarrhea Signs and symptoms • Mild pancreatitis: steady epigastric pain centered near the navel and unrelieved by vomiting • Severe pancreatitis: severe, persistent, piercing abdominal pain, usually in the midepigastric region; may generalize or occur in left upper quadrant and radiate to the back or other areas; typically precipitated by a large meal or alcohol consumption; may improve when patient leans forward or lies on side with knees drawn toward chest • Nausea • Vomiting • Fever • Mild jaundice • Tachycardia • Tachypnea • Muscle spasms • Fatty, foul-smelling stools • Possible hypotension • Grey Turner's sign (flank ecchymosis) • Cullen's sign (periumbilical ecchymosis) • Chvostek's and Trousseau's signs (hypocalcemia) • Abdominal distention, rigidity, and tenderness with hypoactive bowel sounds

Treatment • Maintenance of circulation and fluid volume (typically requires aggressive fluid replacement) • Pain relief (using medications, such as morphine and fentanyl, and positioning) • Reduction of pancreatic secretions to allow pancreas to rest (no oral intake, medications to rest pancreas, and possibly NG tube insertion) • Maintenance of nutrition (nutritional support, such as total parenteral nutrition and enteric feedings) • Prevention or treatment of infection and complications (antibiotic therapy or surgery as needed)

Metabolic and respiratory acidosis and alkalosis • Metabolic acidosis—occurs when poor tissue perfusion allows lactic acid to accumulate • Metabolic alkalosis—occurs with excessive diuretic use, which causes bicarbonate retention • Respiratory alkalosis—occurs early in heart failure when increased respirations cause more carbon dioxide to be blown off and pH to rise • Respiratory acidosis—occurs as heart failure progresses, gas exchange is impaired, and carbon dioxide accumulates

Treatment • Medical emergency • Relief of dyspnea • Improved arterial oxygenation • Diuretics to relieve fluid overload, vasodilators to reduce preload and afterload, or inotropics to increase heart contractility • Possibly surgery for severe heart failure (heart transplant as last resort) • Patient and family education about the disease and its management

Hypophosphatemia • Severe hypophosphatemia: serum phosphorus levels less than 1 mg/dl; may lead to organ failure • Three underlying mechanisms: - Shift of phosphorus into intracellular fluid - Decrease in intestinal absorption - Increased excretion from kidneys • Most common causes: respiratory alkalosis, hyperglycemia, refeeding syndrome, malabsorption syndrome, excessive use of phosphorus-binding antacids, diarrhea, laxative abuse, diuretics, DKA hyperparathyroidism, hypocalcemia, and extensive burns Signs and symptoms • Most commonly occur as a result of severe hypophosphatemia that affects the musculoskeletal, central nervous, cardiac, and hematologic systems • Most common: muscle weakness • Slurred speech, dysphagia, cardiomyopathy, hypotension, decreased cardiac output, rhabdomyolysis, cyanosis, and respiratory failure

Treatment For mild to moderate conditions • Oral supplements • Increased dietary intake For severe conditions • I.V. phosphorus (potassium phosphate or sodium phosphate)

Heat-related health alterations review Heat-related health alterations basics • Develop when the body can't offset rising temperature and retains too much heat • Result from failure of the mechanisms that regulate body temperature • Are easily prevented with adequate hydration Heat transfer • Four methods: conduction, convection, radiation, and evaporation • Body heat lost mainly through radiation and evaporation; if air temperature > 95° F (35° C), evaporation is only means of heat loss Sweat • Body's main way to get rid of extra heat • Heat created by blood flowing through the skin evaporates water from the skin's surface. • Weather and dehydration influence the effectiveness of sweating.

Types of heat-related health alterations • Heat rash (mild) • Heat cramps (mild) • Heat exhaustion (moderate) • Heat syncope (moderate) • Heatstroke (critical)

Fluid balance basics • Fluid movement throughout the body helps maintain body temperature and cell shape. • Fluids help transport nutrients, gases, and wastes. • Most of the body's major organs work together to maintain fluid balance. • The amount of fluids gained through intake must equal the amount lost. Fluid losses • Insensible losses - Immeasurable - Examples: through the skin (affected by humidity and body surface area) and lungs (affected by respiratory rate and depth) • Sensible losses - Measurable - Examples: from urination, defecation, and wounds

Understanding body fluids • Different types of fluids are located in different compartments. • Fluids move throughout the body by going back and forth across a cell's semipermeable membrane. • Distribution of fluids varies with age. Fluid compartments • ICF—fluid inside the cell; must be balanced with ECF • ECF—fluid outside the cell; must be balanced with ICF; made up of 75% interstitial fluid (fluid surrounding the cell) and 25% plasma (liquid portion of blood) • Transcellular fluid—in the cerebrospinal column, pleural cavity, lymph system, joints, and eyes; remains relatively constant

Acid-base basics • Acids—molecules that can give hydrogen molecules to other molecules; include solutions with a pH below 7 • Bases—molecules that can accept hydrogen molecules; include solutions with a pH above 7 • Must maintain a delicate balance for the body to work properly • Metabolism and body functions affected by slight imbalances • Imbalance caused by infection, trauma, and medications

Understanding pH • pH—calculation based on the percentage of hydrogen ions and the amount of acids and bases in a solution • Normal blood pH—7.35 to 7.45, which represents the balance between hydrogen ions and bicarbonate ions Deviation from normal pH • Acidosis—blood pH is below 7.35 and either the hydrogen ion concentration has increased or the bicarbonate level has decreased. • Alkalosis—blood pH is above 7.45 and either the hydrogen ion concentration has decreased or the bicarbonate level has increased. • A pH below 6.8 or above 7.8 is generally fatal. • Deviation compromises well-being, electrolyte balance, activity of critical enzymes, muscle contraction, and basic cellular function.

Burn severity • Requires the use of assessment tools, such as the Rule of Nines or the Lund-Browder classification, to estimate the percentage of body surface area involved • May be categorized as major, moderate, or minor Phases of burns Fluid accumulation phase • Occurs within first 24 to 36 hours after a burn; also known as burn shock phase • Causes fluid to shift from the vascular compartment to interstitial space (third-space shift), resulting in edema • Produces stress hormones that cause the kidneys to retain sodium and water, leading to diminished kidney perfusion and decreased urine output

• Causes such fluid and electrolyte imbalances as: - Hyperkalemia—results from massive cellular trauma, metabolic acidosis, or renal failure; develops as potassium is released into extracellular fluid during initial days following injury - Hypovolemia—results from fluid losses and third-space shift - Hyponatremia—results from increased cellular loss of sodium and water; causes large amounts of sodium to become trapped in edematous fluid - Hypernatremia—can result from aggressive use of hypertonic sodium solutions during fluid replacement therapy - Hypocalcemia—can result when calcium travels to the damaged tissue and becomes immobilized at burn site; may also result from inadequate dietary intake of calcium or inadequate supplementation during treatment - Metabolic acidosis—may result from accumulation of acids released by burned tissue; may also result from decreased perfusion due to hypovolemia

Sodium imbalances review Sodium basics • Major cation in extracellular fluid (90%) • Attracts fluids • Helps transmit impulses in nerve and muscle fibers • Combines with chloride and bicarbonate to regulate acid-base balance • Normal serum level: 135 to 145 mEq/L Sodium balance • Balance is maintained by ADH, which is secreted from the posterior pituitary gland. • The balance depends on what's eaten and how sodium is absorbed in the intestines. • Increased sodium intake results in increased extracellular fluid volume. • Decreased sodium intake results in decreased extracellular fluid volume.

• Increased sodium levels result in increased thirst, release of ADH, retention of water by the kidneys, and dilution of blood. • Decreased sodium levels results in suppressed thirst, suppressed ADH secretion, excretion of water by the kidneys, and secretion of aldosterone to conserve sodium. • Balance is maintained by diffusion, which moves sodium ions into cells and potassium out. • Sodium-potassium pump uses energy to move sodium ions back out of cells and return potassium to cells; it also creates an electrical charge within the cell from the movement of ions, allowing transmission of nerve impulses.


Set pelajaran terkait

MKGT301 Ch. 13-16 (dynamic modules)

View Set

Chapter 14 Coursepoint Pathophysiology

View Set

Chapter 6 - DNA Replication and Repair

View Set